IBPS PO Prelims Exam 2016 Previous Year Paper

IBPS PO 2016 Prelims

Section

Quantitative Aptitude

Questions

25 Questions

Marks

25

Data Interpretation

10 Questions (16 – 25)

10

English

30 Questions (36 – 65)

30

Logical Reasoning

35 Questions (66 – 100)

35

 

Q. 1 What will come in place of question mark (?) in the following number series?

1, 7, 49, 343, ?

A. 16807

B. 1227

C. 2058

D. 2401

E. None of these

 

Q. 2 What will come in place of question mark (?) in the following number series?

13, 20, 39, 78, 145, (?)

A. 234

B. 244

C. 236

D. 248

E. None of these

 

Q. 3 What will come in place of question mark (?) in the following number series?

12 , 35, 81, 173, 357, (?)

A. 725

B. 715

C. 726

D. 736

E. None of these

 

Q. 4 What will come in place of question mark (?) in the following number series?

3, 100, 297, 594, 991, (?)

A. 1489

B. 1479

C. 1478

D. 1498

E. None of these

 

Q. 5 What will come in place of question mark (?) in the following number series?

112, 119, 140, 175, 224, (?)

A. 277

B. 276

C. 287

D. 266

E. None of these

 

Q. 6 What approximate value should come in place of question mark in the following equations.

(6.23/34.98) x 24.999 = ?

A. 560

B. 540

C. 520

D. 580

E. 600

 

Q. 7 What approximate value should come in place of question mark in the following equations.

(4722+3272+5324)/(491+769+132)=?

A. 40

B. 10

C. 15

D. 25

E. 20

 

Q. 8 What approximate value should come in place of question mark in the following equations.

22 (1/3) % of 435.1 – 11/7 of 1734.68 = ?

A. 59

B. 62

C. 50

D. 69

E. 78

 

Q. 9 What approximate value should come in place of question mark in the following equations.

(1681)^1/2 + (4094)^1/3 = ?

A. 57

B. 75

C. 78

D. 67

E. 76

 

Q. 10 What approximate value should come in place of question mark in the following equations.

(803.63)² = ?

A. 506000

B. 787800

C. 646400

D. 767600

E. 566000

 

Questions: 11 – 15

Direction ( 11-15) In each of these questions’ two equations are given. You have to solve these equations and give answer

 

Q. 11 I. x² − 6x = 7

II. 2y² + 13y + 15 = 0

A. if x < y

B. if x > y

C. if x = y and relation cannot be established

D. if x ≥ y

E. if x ≤ y

 

Q. 12 I. 3x² − 7x + 2 = 0

II. 2y² − 11x + 15 = 0

A. if x < y

B. if x > y

C. if x = y and relation cannot be established

D. if x ≥ y

E. if x ≤ y

 

Q. 13 I. 10X² − 7x + 1 = 0

II. 35y² − 12y + 1 = 0

A. if x < y

B. if x > y

C. if x = y or relation cannot be established

D. if x ≥ y

E. if x ≤ y

 

Q. 14 I. 4x² = 25

II. 2y² − 13y + 21 = 0

A. if x < y

B. if x > y

C. if x = y or relation cannot be established

D. if x ≥ y

E. if x ≤ y

 

Q. 15 I. 3x² + 7x = 6

II. 6(2y² + 1) = 17y

A. if x < y

B. if x > y

C. if x = y or relation cannot be established

D. if x ≥ y

E. if x ≤ y

 

Questions: 16 – 20

Refer to the following pie charts and solve the questions based on it.

 

Q. 16 Both the families decide to double the total expenditure keeping the pattern of spending the same as given above. What will be the new ratio of expenditure on food between family

A and family B?

A. 18 : 31

B. 31 : 27

C. 2 : 3

D. 3 : 2

E. None of the above

 

Q. 17 If the total expenses of family B increases three – fold, keeping the expenses on education the same as given above, what will be the expense on education?

A. 6.33%

B. 57%

C. 19%

D. 18%

E. None of the above

 

Q. 18 What will be the expenses on light by family A, as a percentage of expense on light by family B?

A. 120%

B. 83.33%

C. 62.5%

D. 66.66%

E. None of the above

 

Q. 19 If family A and family B decide to combine their expenses, then which one of the following heads will be responsible for the highest expenses?

A. Rent

B. Miscellanous

C. Food

D. Education

E. None of the above

 

Q. 20 In the above question, how many heads will have a lower percentage share in the

combined total expenses of both the families than the percentage share of family B under the same head?

A. 1

B. 2

C. 3

D. 4

E. None of the above

 

Questions: 21 – 25

Study the following bar graph and table carefully to answer the questions given below

The bar graph shows data related to population of different states(in lakhs) in the year 1992 The table shows the ratio b\w male, female and literacy, illiterate and also

graduates and undergraduates.

 

Q. 21 If in the year 1993 there was an increase of 10% population of AP. and 12% of Bihar compared to the previous year, than what was the ratio of the population of AP. to Bihar?

A. 521:540

B. 405:530

C. 408:505

D. 407:560

E. None

 

Q. 22 What was the approximate percentage of women of Andhra Pradesh to the women of HP?

A. 90%

B. 110%

C. 120%

D. 126%

E. 95%

 

Q. 23 If 70% of total no of literate population in Assam are graduate what is the total no of under graduates in the

Assam in the year 1992?

A. 65300

B. 70000

C. 62021

D. 82120

E. None

 

Q. 24 In Haryana, if 70% of the females are literate and 75% of the males are literate, what is the total number of illiterates in the state?

A. 12,2500

B. 85,000

C. 84,000

D. 81,000

E. None

 

Q. 25 What is the ratio of literates in Assam to the literates in Bihar?

A. 2 : 5

B. 3 : 5

C. 7 : 15

D. 2 : 3

E. None

 

Q. 26 Two trains A and B are running in the same direction with speed 60 kmph and 90 kmph respectively. Train B completely crosses a man in Train A in 24sec. The length of the train B is

A. 100

B. 200

C. 250

D. 150

E. None of these

 

Q. 27 Two equal bottles are filled with water and milk. The amount of water in each container is 20% and 30%. What is the ratio of milk in both the bottles respectively ?

A. 3 : 4

B. 5 : 6

C. 8 : 7

D. 9 : 8

E. None of these

 

Q. 28 Ravi bought a car at 30% discount on its original price.He sold it with 50% increase on the price he bought it.The new sale price is by what percent more than the original price ?

A. 2%

B. 4%

C. 8%

D. 5%

E. 1%

 

Q. 29 Among a group of 5 men and 6 women , 4 members is to be selected for an event. Find the probability that at least one man is selected ?

A. 21/22

B. 10/12

C. 8/9

D. 5/4

E. 2/3

 

Q. 30 Two pipes can fill the tank in 4hrs 5hrs respectively while the third pipe can empty the tank in 20hrs, if all the pipes are opened together, then the tank will be filled in

A. 4 (1/2) hrs

B. 2 (1/2) hrs

C. 5 (1/2) hrs

D. 2 (1/4) hrs

E. None of these

 

Q. 31 From the salary, Pavan spent 15% for house rent, 5% for children’s education and 15% for Entertainment. Now he left with Rs.26,000. His salary is

A. 20000

B. 25000

C. 28000

D. 35000

E. 40000

 

Q. 32 A and B invested in a business. They earned some profit which they divided in the ratio of 2:3. If A invested Rs.15,000, the amount invested by B is

A. 21500

B. 20500

C. 22500

D. 23000

E. 25000

 

Q. 33 Milk and water are in the ratio of 3:2 in a mixture of 70 liters. How much water should be added so that the rate of milk and water become 2:3?

A. 20

B. 28

C. 35

D. 40

E. 50

 

Q. 34 Length of a rectangle is reduced by 40%. By what percent would the width have to be increased to maintain the original area?

A. 66.66%

B. 50.55%

C. 44.44%

D. 40.45%

E. 20.22%

 

Q. 35 What approximate value should come in place of the question mark (?) in the following questions. 25.675% of 1321 + 64.328% of 4001= ?

A. 2520

B. 2020

C. 2550

D. 2890

E. 2000

 

Questions: 36 – 40

Read the following passage carefully and answer the questions given below in the context of the passage.

The phenomenon of terrorism signifies violence by disgruntled groups of people determined to achieve certain political goals which they find, are not attainable by lawful means. This menace has lately assumed a global character and many countries including the U.S.A are affected by it. Terrorism is actually an infantile attitude, a sign of immaturity, just as certain other movements indicate political senility. It is wrong, however, to believe that terrorists are born. Like naxalites, Radicals and Reformers, they are the victims of certain unfavourable conditions and circumstances. These circumstances, in turn , are often the redressing political, economic and other grievances. Bitterly disappointed individuals or groups resort to violence in order to focus attention on their problems. Terrorism sprouts and flourishes where there is intense anger and resentment against the administration, the Government’s lapses and postures of neglect towards certain discordant elements feed the unrest, and when this reaches a high level, it bursts into terrorism. Terrorists generally belong to the lower middle class, sometimes to well-placed families and poor class.

 

Q. 36 Terrorism has lately assumed a

A. serious threat

B. global character

C. national character

D. Indian character

E. None of these

 

Q. 37 Like Naxalites, terrorists are the victims of

A. political interference

B. unemployment

C. ignorance

D. unfavourable circumstances and conditions

E. None of these

 

Q. 38 According to the passage, terrorism has affected

A. Asia

B. Britain

C. India

D. the entire world

E. None of these

 

Q. 39 Terrorists belong to

A. the lower middle class

B. well placed families

C. poor class of people

D. all of the above

E. None of these

 

Q. 40 Terrorists are bitterly disappointed individuals who resort to violence in order to

A. dislodge the ruling party

B. get employment

C. focus attention on their problems

D. terrorise the innocent people

E. None of these

 

Questions: 41 – 43

Read the following passage carefully and answer the questions given below it. Once a thief named Kalu had planned to loot the king’s treasury. At midnight, he went to the palace and began to drill a hole in the side wall of the treasury. The king, who was awake in his bedroom just above the treasury, came out to investigate the whirring sound. He was dressed in a simple nightgown and the thief could not recognise him. He asked Kalu who he was and what he was doing. The latter said, “Sir, I am a thief and intend to loot this treasury. I presume that you are also a thief and have come with the same intention. No matter, let us both go inside and we shall share the loot equally. “Both entered the treasury and divided all the money and the jewels equally between them. Inside a locker they found three big diamond pieces. As the thief was puzzled as to how to divide the three pieces into two portions, the king suggested. “We have taken away everything else. Let us leave one diamond piece for the poor king and share the rest equally”. Kalu agreed and when he took his leave, the king asked for his name and address. As Kalu had taken a vow of telling only the truth, he have the correct information. The king took away his share of the loot and hid it in his room. Next morning he asked his Prime Minister to inspect the treasury as he had heard some strange sounds during the previous night. The Prime Minister saw to his horror that all the valuables were missing and only a single diamond was left, perhaps inadvertently, by the thief. He put the diamond in his own shift pocket as its loss could be ascribed to the thief and nobody would suspect the Prime Minister. The Prime Minister went back to the king. The king particularly enquired. “Do you mean that the thief has completely denuded the treasury of its valuables and not a single item has been left?” The Prime Minister confirmed it. The king asked the chief of police to bring in Kalu. When Kalu came he was unable to recognise the king as his accomplice of the previous night. The king asked him, “Are you the thief who has stolen everything from my treasury leaving nothing back?” Kalu confirmed it but said, “Sir, I did leave one diamond back in the locker as advised by an accomplice of mine and it should still be there.” The Prime Minister interrupted saying, “Your Majesty, this thief is lying. There is nothing left in the locker.” The king asked the police chief to search the pockets of the Prime Minister, from where the missing diamond was recovered. The kind told his courtiers, “Here is a Prime Minister, who is a liar and a thief and here is a thief who is at truthful gentleman.”

 

Q. 41 The king came out in the middle of the night in order to

A. Help Kalu to break into the palace treasury

B. Share the loot equally between Kalu and himself

C. Find out the source of and reason for the sound he had heard

D. Catch the thief who had come to steal his valuables

E. None of these

 

Q. 42 Kalu could not recognise the king because

A. The king was wearing clothes like those of an ordinary person

B. The king’s clothes were covered by a simple night down

C. Kalu had never seen the king before

D. Kalu had not seen the king descending from his bedroom

E. None of these

 

Q. 43 Which of the following made the king suspect the Prime Minister? The Prime Minister’s statement that

A. Except for one piece of diamond all other valuables were stolen

B. All the valuables without any exception were stolen from the treasury

C. The thief was lying when he said he had left one diamond back in the locker

D. The search for the diamond did not yield any favourable result

E. None of these

 

Questions: 44 – 50

In the following passage some of the words have been left out, each of which is indicated by a number. Find the suitable word from the options given against each number and fill up the blanks with appropriate words to make the paragraph meaningfully complete.

Let’s think of Jinnah in terms of the history of the subcontinent. Also, once you start to look at him, you see what an ambivalent and complicated figure (44) . Even just to discover his liking for theatre and drama. He (45) with Shakespeare; he wanted to become an actor ; he imitated his hero Chamberlain by wearing a monocle. Sometimes, with his speeches, you don’t know if he believes what he’s saying or if he’s acting. There is a way in which he just took (46) a role. So did he really want what he created ? Did he know what he was doing, or was it almost like a lawyer arguing a brief ? I wanted again to bring him into a human dimension and to show him in relationship to Gandhi, (47) other figures like Tilak. He was a great friend of Tilak’s. Now, that’s a curious friendship. We think of Tilak as a kind of rightist Hindu ; Jinnah defended Tilak in court against the British. His anger at Gandhi was because Gandhi (48) religious! So there are all sorts of twists in Jinnah’s story. There are quite a few people in the world who are fat and are (49).These people are (50) to several kinds of illness. Medical advice to these people is to

 

Q. 44 Find the suitable words given in the options against (44).

A. he was

B. he has been

C. he had been

D. have been

E. as he was

 

Q. 45 Find the suitable words given in the options against (45).

A. had liked

B. was in awe

C. was obsessed

D. related

E. was incorrigible

 

Q. 46 Find the suitable words given in the options against (46).

A. at

B. on

C. in

D. for

E. kind of

 

Q. 47 Find the suitable words given in the options against (47).

A. at par

B. in relation to

C. with

D. as in

E. as of

 

Q. 48 Find the suitable words given in the options against (48).

A. was being

B. had been

C. has been

D. could have been

E. would be

 

Q. 49 Find the suitable words given in the options against (49).

A. underweight

B. overweight

C. physically fit

D. thin

E. emaciated

 

Q. 50 Find the suitable words given in the options against (50).

A. susceptible

B. unsusceptible

C. safe

D. unsafe

E. cured

 

Questions: 51 – 55

Given below is a sentence marked (A), after which there are five more sentences marked, (B), (C), (D), (E) and (F). These five sentences are not in their proper sequence.

Rearrange them so as to form a meaningful paragraph with (A) as the first sentence; then answer the questions given below them.

(A) when I reached the station, the train had already arrived

(B) One came running up to me and with his help.I began pilling up my luggage on the entrance of the compartment.

(C) There were five minutes left for its departure.

(D) However, I made an effort to remain cool and called out to a coolie.

(E) As I carried up the last item, the train began to crawl out of the station.

(F) This made me a bit nervous as I had a lot of luggage with me.

 

Q. 51 Which sentence should come third in the passage?

A. A

B. C

C. F

D. E

 

Q. 52 Which sentence should come second?

A. C

B. A

C. D

D. F

 

Q. 53 Which sentence should come fifth?

A. C

B. B

C. E

D. F

 

Q. 54 Which sentence should come fourth?

A. B

B. D

C. A

D. E

 

Q. 55 Which sentence should come last?

A. C

B. E

C. A

D. B

 

Questions: 56 – 65

Read each sentence to find out whether there is any grammatical or idiomatic error in it. The error if any, will be in one part of the sentence, i.e.in (A), (B), (C) or (D). Do not look for errors in spelling and punctuation. When you find an error in a sentence, mark the incorrect part. If there is no error, mark (E).

 

Q. 56 Find the Spot Grammatical Error:

A. My father told me (A)

B. that if I do not (B)

C. study seriously (C)

D. I would fail. (D)

E. No error. (E)

 

Q. 57 Find the Spot Grammatical Error:

A. The girl who (A)

B. met you (B)

C. yesterday (C)

D. is my cousin sister. (D)

E. No error (E)

 

Q. 58 Find the Spot Grammatical Error:

A. The teacher asked (A)

B. the student (B)

C. why had he (C)

D. not done his homework . (D)

E. No error (E)

 

Q. 59 Find the Spot Grammatical Error:

A. I want (A)

B. to get rid from (B)

C. this bad habit (C)

D. immediately. (D)

E. No error (E)

 

Q. 60 Find the Spot Grammatical Error:

A. The news that (A)

B. the Lebanon Prime Minister (B)

C. was killed (C)

D. in aircrash is correct. (D)

E. No error (E)

 

Q. 61 Find the Spot Grammatical Error:

A. Be careful (A)

B. lest you will (B)

C. fall (C)

D. and hurt yourself. (D)

E. No error (E)

 

Q. 62 Find the Spot Grammatical Error:

A. I, along with my parents (A)

B. were (B)

C. present (C)

D. at the reception. (D)

E. No error (E)

 

Q. 63 Find the Spot Grammatical Error:

A. Although I tried (A)

B. my best (B)

C. I could not prevent him to do (C)

D. that mischief. (D)

E. No error (E)

 

Q. 64 Find the Spot Grammatical Error:

A. Many persons (A)

B. who came to attend the meeting (B)

C. did not know (C)

D. each other. (D)

E. No error (E)

 

Q. 65 Find the Spot Grammatical Error:

A. The telephone bell (A)

B. rang (B)

C. when I (C)

D. took bath. (D)

E. No error (E)

 

Q. 66 Statements :

S ≤R ≥ Y ≤ M; Y ≥ L < E

Conclusion I : M > S

Conclusion ll: Y>S

Conclusion III: E < R

Conclusion IV: L=S

A. None is true

B. Only I is true

C. Only III is true

D. Only II is true

E. Only I and IV are true

 

Q. 67 Statements :

K > S ≥T = N ≤ B < U ≥ M

Conclusion I: K > B

Conclusion II: N < M

Conclusion III : S ≥ U

Conclusion IV : N < K

A. Only II and IV are true

B. Only I and II are true

C. None is true

D. Only IV is true

E. Only II and III are true

 

Q. 68 Statements :

K < Y > B ≤ U; B ≥ F ≥ M

Conclusion I: U > M

Conclusion II : U > K

Conclusion III: M = U

Conclusion IV : Y > M

A. Only either II or III is true

B. Only IV is true

C. Only IV and either I or III are true

D. Only III is true

E. Only II is true

 

Q. 69 Statements :

T > S ≥ U = J≤ D ≤ Y < H

Conclusion I: T > J

Conclusion II: H > U

Conclusion III : U ≤ Y

Conclusion IV : S > D

A. Only I, II and III are true

B. Only I and either III or IV are true

C. All I, II, III and IV are true

D. Only II is true

E. Only I and II are true

 

Q. 70 Statements :

P > B ≤ Z < A; D ≥ B > T

Conclusion I: P > T

Conclusion II : T < A

Conclusion III: D > T

Conclusion IV: A > B

A. Only I and III are true

B. Only III is true

C. All are true

D. Only IV is true

E. Only I is true

 

Questions: 71 – 75

In each question below are given two or three statements followed by two or three conclusions numbered I, II and III. You have to take the given statements to be true even if they seem to be at variance with commonly known facts and then decide which of the given conclusions logically follows from the given statements, disregarding commonly known facts. Give Answer.

 

Q. 71 Statements:

Some Boys are Girls.

All Boys are Madam.

Conclusions:

I. Those Boys which are not Girls are also Madam.

II. Those Boys which are not Girls are not necessarily Madam.

A. Only Conclusion I follows.

B. Only Conclusion II follows.

C. Both I & II follow.

D. Either I or II follow.

E. None follows.

 

Q. 72 Statements:

All Cows are Milk.

No Milk is White.

Conclusions:

I. Some Cows are White.

II. No Cow is White.

A. Either I or II follow.

B. Only Conclusion I follows.

C. Both I & II follow.

D. Only Conclusion II follows.

E. None follows.

 

Q. 73 Statements:

Some Pens are Pen-drive.

All Printers are Pen-drive.

Conclusions:

I. All Pens being Printers is a possibility.

II. No Printer is a Pen.

A. Only Conclusion II follows.

B. Both I & II follow.

C. Only Conclusion I follows.

D. Either I or II follow.

E. None follows.

 

Q. 74 Statements:

No Stone is a Metal.

Some Metals are Paper.

All Papers are Glass.

Conclusions:

I. All Stones being Glass being is a possibility.

II. No Stone is a Paper.

A. Only Conclusion I follows.

B. Only Conclusion II follows.

C. Both I & II follow.

D. Either I or II follow.

E. None follows.

 

Q. 75 Statements:

Some Apples are Cake.

Some Cakes are Candle.

Conclusions:

I. All Cakes are Apple.

II. Some Apples are Candle.

III. No Apple is a Candle.

A. Either I or II follow.

B. Only Conclusion I follows.

C. Both I & II follow.

D. Only Conclusion II follows.

E. None follows.

 

Questions: 76 – 80

Study the following information carefully and answer the given questions. Eight family members Samavesh, Trisuram, Ubaida, Vikram, Walida, Xander, Yashika and Zareen are sitting around a circular table but not necessarily in the same order. Some of them are females and some are males. All of them are related to each other in the same way or the other. Some of them are facing the centre while some are facing outside (i.e. opposite to the centre).

1. Only two people sit between Trisuram and Walida. Trisuram faces the centre.

2. Xander sits second to the right of Trisuram. Walida is the wife of Samavesh. No

females is an immediate neighbour of Walida

3. Ubaida is not an immediate neighbor of Trisuram. Ubaida is the daughter of Walida. Both the immediate neighbours of Ubaida face the centre.

4. Only three people sit between Samavesh and Ubaida’s brother. Xander is not the brother of Ubaida. Neither Samavesh nor Ubaida’s brother is an immediate neighbour of Xander.

5. Zareen, the wife of Trisuram, sits to the immediate left of Vikram. Both Yashika and Samavesh face a direction opposite to that of Ubaida (i.e. Ubaida faces the center then both Yashika and Samavesh face outside and vice – versa).

6. Ubaida’s husband sits second to the left of Yashika. Trisuram’s father sits to the immediate right of Walida.

7. Trisuram sits second to the right of Samavesh’s father. Both the immediate neighbours of Xander are females.

 

Q. 76 How many people sit between Trisuram and Samavesh’s father when counted from the right of Trisuram?

A. Four

B. Three

C. None of the above

D. One

E. Two

 

Q. 77 Who amongst the following sits exactly between Yashika and Walida when counted from the left of Yashika?

A. Trisuram

B. Xander

C. Samavesh

D. Zareen

E. Ubaida

 

Q. 78 Which of the following statements regarding Trisuram is definitely true?

A. Xander and Zareen are immediate neighbors of Trisuram

B. Trisuram sits second to the left of Xander

C. Trisuram is the son of Samavesh

D. None of the given options is correct

E. Vikram is the father of Trisuram

 

Q. 79 Who amongst the following faces outside (i.e. opposite to the centre)?

A. Walida

B. Vikram

C. Ubaida

D. Zareen

E. Trisuram

 

Q. 80 If it is given that Yashika is married to Xander, then what is the position of Trisuram with respect to Yashika’s daughter – in – Law?

A. Third to the Right

B. Second to the Right

C. Immediate Right

D. Second to the Left

E. Third to the Left

 

Questions: 81 – 85

Study the following information carefully to answer the given questions.

There are eight friends, namely Suresh, Rohit, Hrithik, Swades, Farhan, Jiten, Gopal, and Emran. All of them own a different 4-wheeler, namely Maruti Suzuki, Tata Nano, Nissan Datsun, Honda City, Mercedes Benz, Indigo, Santro and BMW, though not necessarily in the same order. All of them are sitting around a circular table facing the centre. Emran is not a neighbour of Rohit while Jiten owns neither Santro nor BMW. There is only one person between Suresh and Swades. Hrithik, who owns Indigo, is third to the left of the one who owns Nissan Datsun. The one who is sitting opposite the one who has Tata Nano owns BMW. Neither Suresh nor Swades has Nissan Datsun. There are two persons between Farhan and Gopal, who own Tata Nano and Mercedes Benz respectively, but neither of those two persons is Hrithik. Suresh is second to the left of Farhan and owns Honda City. Rohit is an immediate neighbour of Gopal.

 

Q. 81 Who among the following owns Santro?

A. Emran

B. Swades

C. Gopal

D. Jiten

E. Can’t be determined

 

Q. 82 How many persons are there between Hrithik and Gopal?

A. One

B. Two

C. Three

D. Four

E. None of these

 

Q. 83 Which of the following 4-wheelers is owned by Swades?

A. Mercedes Benz

B. Santro

C. BMW

D. Maruti Suzuki

E. None of these

 

Q. 84 Who is sitting third to the right of the person possessing Maruti Suzuki?

A. Suresh

B. Rohit

C. Emran

D. Swades

E. Gopal

 

Q. 85 Which of the following is not true about Jiten?

A. He is sitting exactly between Farhan and Rohit.

B. He is sitting opposite the person who owns Santro.

C. He owns Maruti Suzuki.

D. None is true.

E. All are true.

 

Questions: 86 – 90

Read the following information carefully and answer the questions given below it —

Six exams Maths, Science, History, Economics, English and Hindi are to be scheduled starting from 2nd March and ending on 8th march wit Sunday being an off day, not necessarily in the same order. Each of the exam has different time duration: 40 mins, 50 mins, 60 mins, 75 mins, 90 mins and 100 mins, again not necessarily in the same order. 8th March is not Sunday and an exam of 40 mins is scheduled on that day. Maths exam is for less than 60 mins and is scheduled immediately before English exam. There are two exams scheduled between Hindi exam which is for 100 mins and History exam which is for 60 mins. English exam is before Sunday and there are two days between Sunday and maths exam. Economics exam which is for 75 mins is not scheduled on 2nd March. The exam scheduled on Saturday is of 100 mins

 

Q. 86 How many exams are scheduled before Sunday?

A. Two

B. One

C. Five

D. Three

E. None of these

 

Q. 87 Which of the following combinations of exam – Day – Time Duration is correct ?

A. English – Wednesday – 75 mins

B. Maths – Thursday – 50 mins

C. History – Thursday – 60 mins

D. Hindi – Tuesday – 100 mins

E. None is correct

 

Q. 88 What is the time duration of Science exam?

A. 90 mins

B. 75 mins

C. 50 mins

D. 40 mins

E. None of these

 

Q. 89 On which day is Economics exam scheduled?

A. Monday

B. Saturday

C. Tuesday

D. Friday

E. Cannot be determined

 

Q. 90 Which day is Sunday?

A. 3rd March

B. 2nd March

C. 5th March

D. 6th March

E. Cannot be determined

 

Questions: 91 – 95

Read the following information and answer the questions that follow:

(1) There are seven persons A, B, C, D, E, F and G based in Delhi. Each of them is from a different state, has a different profession, and plays a different instrument.

(2) C, a doctor, is from Bihar.

(3) E and F play mandolin and violin, though not necessarily in that order.

(4) A is not from Kerala.

(5) The person from Kerala is an engineer and plays guitar.

(6) The lawyer plays sitar.

(7) The businessman from UP plays violin.

(8) The teacher and the cricketer play flute and piano, though not necessarily in that order.

(9) F is a pilot.

(10) The Maharashtrian is a teacher.

(11) The Gujarati plays piano.

(12) G, a Punjabi, does not play sarod.

(13) B is a cricketer.

 

Q. 91 Which state does A belong to?

A. Gujarat

B. Kerala

C. Maharashtra

D. Can’t say

E. None of these

 

Q. 92 Which instrument does B play?

A. Flute

B. Piano

C. Sarod

D. Can’t say

E. Sitar

 

Q. 93 Which instrument does C play?

A. Mandolin

B. Sitar

C. Violin

D. None of these

E. Sarod

 

Q. 94 What is D’s profession?

A. Engineer

B. Lawyer

C. Teacher

D. Can’t say

E. Cricketer

 

Q. 95 Which state does E belong to?

A. Gujarat

B. Kerala

C. UP

D. Can’t say

E. None of these

 

Q. 96 A is D’s brother is B’s father. B and C are sisters. How is C related to A?

A. Cousin

B. Niece

C. Aunt

D. Nephew

E. None of these

 

Q. 97 If X is the brother of the son Y’s son., then how is X related to Y?

A. Son

B. Brother

C. Nephew

D. Grandson

E. None of these

 

Q. 98 Pointing towards a girl in the picture, Sunita said, “She is the mother of Renu whose father is my son.” How Sunita is related to that girl in the picture?

A. Mother

B. Aunt

C. Cousin

D. Data inadequate

E. None of the above

 

Q. 99 A man said to a woman, “Your only brother’s son is my wife’s brother.” How is the woman related to the man’s wife?

A. Aunt

B. Sister

C. Mother

D. Brother

E. None of these

 

Q. 100 A and B form a married couple. X and Y are the brothers. X is the brother of A. How is Y related to B?

A. Brother

B. Cousin

C. Son-in-law

D. Brother-in-law

E. None of these

 

 

Answer Sheet
Question 1 2 3 4 5 6 7 8 9 10
Answer D D A E C D B D A C
Question 11 12 13 14 15 16 17 18 19 20
Answer B A D A C A A B C B
Question 21 22 23 24 25 26 27 28 29 30
Answer D D B A C B C D A B
Question 31 32 33 34 35 36 37 38 39 40
Answer E C C A D B D D D C
Question 41 42 43 44 45 46 47 48 49 50
Answer C B B A C B B A B A
Question 51 52 53 54 55 56 57 58 59 60
Answer C A B B B B D C B E
Question 61 62 63 64 65 66 67 68 69 70
Answer B B C D D A D C A C
Question 71 72 73 74 75 76 77 78 79 80
Answer A D C A E D D C C E
Question 81 82 83 84 85 86 87 88 89 90
Answer A C B D E E B D A D
Question 91 92 93 94 95 96 97 98 99 100
Answer C B E A C B D E A D

IBPS PO Prelims Exam 2015 Previous Year Paper

IBPS PO 2015 Prelims Exam

Date :03-10-2015

Q : 1 – 5

Directions (1-5) : In the following sentences, there are two blanks for which options have been given. Choose the most appropriate option.

 

 

Q. 1 The proposal has been ………………. and will be sent to the cabinet for final…………

A. designed, process

B. drafted, approval

C. amend, sanction

D. said, objection

E. mentioned, figure

 

Q. 2 The fake diesel factory was being …………… under the ……………… of a diary.

A. operated, operation

B. captured, array

C. functioned, alias

D. run, grab

E. activated, control

 

Q. 3 The moment the staff opened the office, unidentified ………………, ……………… weapons entered and asked the cashier to hand over the keys to the vault.

A. miscreants, brandishing

B. object, carrying

C. persons, had

D. people, associating

E. man, lifting

 

Q. 4 The singer, who has been very ……………… about his faith in the party , also ……………… at the grand event.

A. vocal, performed

B. strong, sang

C. open, dance

D. loud, came

E. creative, present

 

Q. 5 Over the years, the town has ……………… popularity as the best ……………… for paragliding and pilots from across the world visit it.

A. acquired, spots

B. claim, one

C. gained, destination

D. sought, wonder

E. sent, place

 

Questions: 6 – 10

Directions (6-10) : Read these sentences to find out whether there is any grammatical error in them. The error, if any, will be in one part of the sentence.

Mark the part with the error as your answer. If there is no error, mark ‘No error’ as your answer. (Ignore the errors of punctuation, if any.)

 

Q. 6 Many citizens are gravitating towards the nation’s second-largest State because it offer ample job opportunities.

A. Many citizens are gravitating

B. towards the nation’s

C. second-largest State because it offer

D. ample job opportunities

E. No error

 

Q. 7 Most African nations were largely shielded from the 2008 financial crisis by China’s insatiable demand for natural resources.

A. Most African nations were largely

B. shielded from the 2008 financial crisis

C. by China’s insatiable demand

D. for natural resources

E. No error

 

Q. 8 Skeptics worries that the devaluation of the country’s currency is a desperate move to bail out struggling exporters.

A. Skeptics worries that the devaluation

B. of the country’s currency is

C. a desperate move to

D. bail out struggling exporters

E. No error

 

Q. 9 Consumers are constantly been encouraged to take advantage of the lowered interest rates.

A. Consumers are constantly been

B. encouraged to take

C. advantage of the

D. lowered interest rates

E. No error

 

Q. 10 Emerging economies are dominating the news but for all the wrong reasons.

A. Emerging economies are

B. dominating the news

C. but for

D. all the wrong reasons

E. No error

 

Questions: 11 – 20

Directions (11-20) : Read the following passage carefully and answer the questions given below it. Certain words/phrases are given in quotes to help you locate them while answering some of the questions.

Over the past few days alone, the China’s Central bank has pumped extra cash into the financial system and cut interest rates. The aim is to “free” more cash for banks to lend and provide a boost for banks seeking to improve the return on their assets. The official data though, suggested that bad loans makeup only 1.4% of their balance sheets. How to explain the discrepancy? One possible answer is that bad loans are a “lagging” indicator i.e., it is only after the economy has struggled for a while that borrowers began to suffer. Looked at this way, China is trying to anticipate problems keeping its bank in good health by sustaining economic growth of nearly 7% year-on-year. Another more worrying possibility is that bad loans are worse than official data indicate. This does not look to be the cause for China’s biggest banks, which are managed conservatively and largely focus on the country’s biggest value and quality borrowers. But, there is “mounting” evidence that when it comes to smaller banks, especially those yet to list on the stock market, bad loans piling up. That is important because unlisted lenders account for just over a third of the Chinese banking sector, making them as big as Japan’s entire banking industry. Although, non-performing loans have edged up slowly, the increase in specialmention loans (a category that includes those overdue but not yet classified as impaired loans) has been much bigger. Special mention loans are about 2% at most of China’s big listed banks, suggesting that such loans must be much higher at their smaller, unlisted peers. Many of these loans are simple bad debts which banks have not yet admitted to. Another troubling fact is that fifteen years ago, the government created asset management companies (often referred to as bad banks) to take on the non performing loans of the lenders. After the initial transfer these companies had little to pay. But, last year, Cinda, the biggest of the bad banks, bought nearly 150 billion Yuan ($ 24 billion) of distressed assets last year, two-thirds more than in 2013. These assets would have raised the banks bad-loan ratio by a few tenths of a percentage point. Although such members do not seem very alarming, experts who reviewed last year’s result for 158 banks, of which only 20 are listed found that ‘shadow loans’, loans recorded as investment which maybe a disguise for bad loans have grown to as much as 5.7 billion Yuan, or 5% of the industry’s assets. These are heavily concentrated on the balance sheets of smaller-unlisted banks, and at the very least, all this “points” to a need for recapitalisation of small banks.

 

Q. 11 Choose the word which is most nearly the same in meaning to the word “Lagging” given in quotes as used in the passage.

A. Delayed

B. Breaking

C. Stopped

D. Protecting

E. Tying

 

Q. 12 Choose the word which is opposite in meaning to the word “Free” given in quotes as used in the passage.

A. Expensive

B. Secret

C. Complimentary

D. Restrict

E. Charged

 

Q. 13 According to the passage which of the following can be said about China’s large banks?

A. These are cautiously run.

B. Their clients are mainly high value.

C. 2% of their loans have been classified as overdue but not impaired.

A. only B

B. only A

C. All A, B and C

D. A and C

E. B and C

 

Q. 14 Which of the following is the central idea of the passage?

A. Small banks should be permitted to become listed on the stock exchange.

B. The government should be away with asset management companies.

C. China’s financial crisis is not as serious as it is being made out to be.

D. China’s Central Bank has failed to predict and stop the decline of its banks.

E. There is trouble brewing in China’s small unlisted banks.

 

Q. 15 Choose the word which is most nearly the same in meaning to the word “Points” given in quotes as used in the passage.

A. Peaks

B. Moments

C. Arguments

D. Indicates

E. Plugs

 

Q. 16 Which of the following is true in the context of the passage?

A. China has not implemented any resources to help its banking sector in recent times.

B. Approximately 32% of China’s banking sector is unlisted.

C. China’s stock market has plummeted in recent times.

D. China’s banking industry is experiencing a boom unlike that of China.

E. None of the given options is true in the context of the passage

 

Q. 17 What does the example of Cinda convey?

A. Many of the loans given by China’s banks are in trouble.

B. Many such large Chinese asset management companies are failing.

C. China’s economy is overly dependent on large banks.

D. China is the ideal destination for small banks to flourish.

E. Such companies have become obsolete.

 

Q. 18 Choose the word which is opposite in meaning to the word “Mounting” given in quotes as used in the passage.

A. Melting

B. Accumulating

C. Removing

D. Submerging

E. Decreasing

 

Q. 19 Which of the following best describes experts’ findings regarding shadow loans?

A. Shadow loans have been steadily falling and are negligible at present.

B. These are growing substantially and indicate the need for reform of small banks.

C. Shadow loans are unfairly being passed on to asset management companies.

D. These loans are inconsequential for the health of banks.

E. The findings are faulty as these include only a few listed banks.

 

Q. 20 What is the author’s view regarding small banks?

A. These have a better loan portfolio then large banks.

B. These are in good health helping to sustain economic growth of 7%.

C. These should be merged with large banks to bail them out of trouble.

D. Regulations governing these banks should be relaxed.

E. Other than those given as options.

 

Questions: 21 – 25

Directions (21-25) : Rearrange the given six sentences/group of sentences A,B,C,D,E and F in a proper sequence so as to form a meaningful paragraph and then answer the given questions.

A. Global investors are quaking over the prospect of a devastating slump in the world’s second biggest economy.

B. A possible answer could be that the country’s troubles raise doubts about whether its policymakers have the tools to keep their economy growing at a healthy place something that has been a constant reassurance for more than two decades.

C. And they are fast loosing confidence that the country’s policymakers, seemingly so sure-footed in the past, know how to solve the problem.

D. However, such a domino effect is significant but hardly catastrophic so why the hysteria?

E. China is exporting something new to the world economy fear.

F. Apart from the shrinking confidence, the biggest fear is that a collapsing

Chinese economy would derail others around the world – from emerging markets in Chile and Indonesia to industrial powers such as the United States.

 

Q. 21 Which of the following should be the SECOND sentence after rearrangement?

A. A

B. B

C. F

D. D

E. E

 

Q. 22 Which of the following should be the FIRST sentence after rearrangement?

A. A

B. C

C. B

D. F

E. E

 

Q. 23 Which of the following should be the SIXTH (LAST) sentence after rearrangement?

A. E

B. D

C. A

D. B

E. F

 

Q. 24 Which of the following should be the FIFTH sentence after rearrangement?

A. A

B. D

C. E

D. F

E. C

 

Q. 25 Which of the following should be the FOURTH sentence after rearrangement?

A. A

B. B

C. C

D. F

E. D

 

Questions: 26 – 30

Directions (26-30) : In the following passage, there are blanks, each of which has been numbered. Against each, five words are suggested, one of which fits the blank appropriately. Find out the appropriate word in each case.

Primary school enrollment in India has been a success story. ……..26……… due to various programmes and drives to increase enrollment even in remote areas.

With enrollment reaching at least 96% since 2009 and girls ……27…….. up 56% of new students between 2007 and 2013, it is clear that many …..28…… of access to schooling have been …..29….. Improvement in infrastructure has been the ….30…. behind achieving this and now in India 98% habitations have a primary school within 1 km and 92% have an upper primary school within a 3 km walking distance.

 

Q. 26 Find out the appropriate one from the options

A. Mostly

B. Properly

C. Totally

D. Optionally

E. Largely

 

Q. 27 Find out the appropriate one from the options

A. coming

B. reaching

C. counting

D. making

E. touching

 

Q. 28 Find out the appropriate one from the options

A. issue

B. opportunities

C. problems

D. efforts

E. exertions

 

Q. 29 Find out the appropriate one from the options

A. accustomed

B. addressed

C. met

D. forwarded

E. dissolved

 

Q. 30 Find out the appropriate one from the options

A. main

B. effort

C. force

D. compulsion

E. awareness

 

Questions: 31 – 35

Directions (31-35) : Refer to the table and answer the given questions.

Note : Few values are missing in the table (indicated by -). A candidate is expected to calculate the missing value, if it is require to answer the given questions on the basis of given data and information.

 

Q. 31 Out of the number of qualified candidates from state P in 2008, the respective ratio of male and female candidate is 11 : 7. If the number of female qualified candidates from state P in 2008 is 126, what is the number of appeared candidates (both male and female) from state P in 2008?

A. 630

B. 510

C. 570

D. 690

E. 540

 

Q. 32 Number of appeared candidates from state Q increased by 100% from 2006 to 2007. If the total number of qualified candidates from state Q in 2006 and 2007 together is 408, what is the number of appeared candidates from state Q in 2006?

A. 380

B. 360

C. 340

D. 320

E. 300

 

Q. 33 What is the difference between number of qualified candidates from state P in 2006 and that in 2007?

A. 12

B. 22

C. 14

D. 24

E. 16

 

Q. 34 If the average number of qualified candidates from state Q in 2008, 2009 and 2010 is 2010, what is the number of qualified candidates from state Q in 2010?

A. 191

B. 195

C. 183

D. 187

E. 179

 

Q. 35 If the respective ratio between number of qualified candidates from state P in 2009 and 2010 is 14 : 9, what is the number of qualified candidates from state P in 2010?

A. 252

B. 207

C. 216

D. 234

E. 198

 

Questions: 36 – 40

Directions (36-40) : What should come in place of question mark (?) in the following questions?

Q. 36 √575 ÷ ? × (14.98)^2 = 450

A. 15

B. 10

C. 7

D. 4

E. 12

 

Q. 37 (30.01)^2 − (19.98)^2 − ?=(21.81)^2

A. 49

B. 50

C. 16

D. 39

E. 41

 

Q. 38  820.15 + 2379.85 + 140.01 × 4.99 = ?

A. 4400

B. 3900

C. 3000

D. 4000

E. 4300

 

Q. 39   39.97% of 649.8 ÷ 13.05 = 45.12 − ?

A. 40

B. 15

C. 25

D. 10

E. 30

 

Q. 40 (674.87 + 59.98) ÷ 35.02 = ?

A. 29

B. 27

C. 19

D. 21

E. 11

 

Questions: 41 – 45

Directions (41-45) : Refer to the graph and answer the following questions.

 

Q. 41 The number of people who travelled by train B on Friday is 20% more than the people who travelled by the same train on Thursday. What is the respective ratio between the number of people who travelled on Friday and those who travelled on Saturday by the same train?

A. 4 : 5

B. 3 : 4

C. 5 : 6

D. 3 : 5

E. 1 : 4

 

Q. 42 What is the difference between the total number of people who travelled by train B on Monday and Tuesday together and the total number of people who travelled by train A on Saturday and Sunday together?

A. 200

B. 230

C. 210

D. 250

E. 240

 

Q. 43 What is the average number of people travelling by train A on Monday, Tuesday,

Wednesday and Thursday?

A. 220

B. 190

C. 205

D. 195

E. 210

 

Q. 44 The number of people who travelled by train A decreased by what percent from Saturday to Tuesday?

A. 35%

B. 40%

C. 30%

D. 42%

E. 33%

 

Q. 45 The total number of people who travelled by both the given trains together on Sunday is approximately what percent more than the total number of people who travelled by both the given trains together on Wednesday?

A. 128%

B. 123%

C. 142%

D. 118%

E. 135%

 

Q. 46 Rs 6,100/- was partly invested in Scheme a at 10% p.a. compound interest (compounded annually) for 2 years and partly in Scheme B at 10% p.a. simple interest for 4 years. Both the schemes pay equal interests. How much was invested in Scheme A?

A. Rs 3,750/-

B. Rs 4,500/-

C. Rs 4,000/-

D. Rs 3,250/-

E. Rs 5,000/-

 

Q. 47 ‘A’ bought a certain quantity of oranges at total cost of Rs 1200/-. He sold 1/3rd of those oranges at 20% loss. If A earns an overall profit of 10%, at what percent profit did A sell the rest of the oranges?

A. 16%

B. 15%

C. 22%

D. 25%

E. 20%

 

Q. 48 Present age of Bob is equal to Abby’s age 8 years ago. Four years hence, the respective ratio between Bob’s age and Abby’s age will be 4 : 5 at that time. What is Bob’s present age?

A. 24 years

B. 32 years

C. 40 years

D. 20 years

E. 28 years

 

Q. 49 Two equations numbered 1 and 2 are given. You have to solve both the equations and mark the appropriate option.

(1) 2x^2 + 19x + 45=0

(2) 3x^2 – 13x + 12=0

A. if x > y

B. if x ≥ y

C. if x < y

D. If relationship between x and y cannot be determined

E. if x ≤ y

 

Q. 50 Two equations numbered 1 and 2 are given. You have to solve both the equations and mark the appropriate option.

(1) 3x^2 – 13x + 12 = 0

(2) 2y^2 – 15y + 28 = 0

A. if x > y

B. if x ≥ y

C. if x < y

D. If relationship between x and y cannot be determined

E. if x ≤ y

 

Q. 51 Two equations numbered 1 and 2 are given. You have to solve both the equations and mark the appropriate option.

(1) x^2 = 16

(2) 2y^2 – 17y + 36 = 0

A. if x > y

B. if x ≥ y

C. if x < y

D. If relationship between x and y cannot be determined

E. if x ≤ y

 

Q. 52 Two equations numbered 1 and 2 are given. You have to solve both the equations and mark the appropriate option.

(1) 6x^2 + 19x + 15 = 0

(2) 3y^2 + 11y + 10 = 0

A. if x > y

B. if x ≥ y

C. if x < y

D. If relationship between x and y cannot be determined

E. if x ≤ y

 

Q. 53 Two equations numbered 1 and 2 are given. You have to solve both the equations and mark the appropriate option.

(1) 2x^2 – 11x + 15 = 0

(2) 2y^2 – 11y + 14 = 0

A. if x > y

B. if x ≥ y

C. if x < y

D. If relationship between x and y cannot be determined

E. if x ≤ y

 

Q. 54 A started a business. After 4 months from the start of the business, B and C joined. The respective ratio between the investments of A, B and C was 4 : 6 : 5. If A’s share in annual profit was Rs 250/- more than C’s share, what was the total annual profit earned?

A. Rs 3,740/-

B. Rs 3,910/-

C. Rs 4,250/-

D. None of these

E. Rs 3,450/-

 

Q. 55 A person has to travel from point A to point B in certain time. Travelling at a speed of 5 km/h he reaches 48 min late and while traveling at a speed of 8 km/h he reaches 15 min early. What is the distance from point A to B?

A. 15 km

B. 9 km

C. 12 km

D. 18 km

E. 14 km

 

Q. 56 28 men can complete a piece of work in 15 days and 15 women can complete the same piece of work in 24 days. What is the respective ratio between the amount of work done by 30 men in 1 day and the amount of work done by 18 women in 1 day?

A. 4 : 7

B. 9 : 15

C. 3 : 5

D. 2 : 6

E. None of these

 

Q. 57 18 litre of pure water was added to a vessel containing 80 litre of pure milk. 49 litre of the resultant mixture was then sold and some more quantity of pure milk and pure water was added to the vessel in the respective ratio of 2 : 1. If the resultant respective ratio of milk and water in the vessel was 4 : 1, what was the quantity of pure milk added in the vessel? (in litre)

A. 4 litre

B. 8 litre

C. 10 litre

D. 12 litre

E. 2 litre

 

Q. 58 A certain sum is divided among A, B and C in such a way that A gets Rs 40/- more than the 1/2 of the sum. B gets Rs 120/- less than 3/8th of the sum and C gets Rs 200/-. What is the total sum?

A. Rs 1,100/-

B. Rs 850/-

C. Rs 960/-

D. Rs 1,200/-

E. None of these

 

Questions: 59 – 63

Directions (59-63) : What will come in place of question mark (?) in the given number series?

 

Q. 59 123 140 106 157 89 ?

A. 214

B. 139

C. 198

D. 169

E. 174

 

Q. 60    190 94 46 22 ? 4

A. 19

B. 15

C. 10

D. 8

E. 16

 

Q. 61    320 320 314 290 230 ?

A. 114

B. 110

C. 50

D. 98

E. 142

 

Q. 62    3 4 9 28 113 ?

A. 782

B. 424

C. 646

D. 384

E. 566

 

Q. 63    8 4 6 15 ? 236.25

A. 64.5

B. 84

C. 52.5

D. 36

E. 46

 

Q. 64 The respective ratio of curved surface area and total surface area of a cylinder is 4 : 5. If the curved surface area of the cylinder is 1232 (cm)^2, what is the height? (in cm)

A. 14 cm

B. 28 cm

C. 7 cm

D. 56 cm

E. 24 cm

 

Q. 65 A bag contains 3 red balls, 5 yellow balls and 7 pink balls. If one ball is drawn at random from the bag, what is the probability that it is either pink or red?

A. 1/7

B. 2/3

C. 4/9

D. 5/7

E. None of these

 

Questions: 66 – 67

Directions (66-67) : Study the following information to answer the given questions.

R is married to U. U is the mother of L. L is the sister of D. U has only one daughter. D is married to J. K is the son of J. F is the mother of J.

 

Q. 66 How is D related to F?

A. Cannot be determined

B. Daughter

C. Daughter-in-law

D. Son-in-law

E. Son

 

Q. 67 How is R related to K?

A. Cannot be determined

B. Father-in-law

C. Grandmother

D. Grand father

E. Uncle

 

Questions: 68 – 72

Directions (68-72) : Study the following information to answer the given questions.

Seven people- P, Q, R, S, T, U and V have a seminar but not necessarily in the same order, on seven different months (of the same year) namely January, February, March, June, August, October and December. Each of them also likes a different fruit namely Banana, Grapes, Papaya, Orange, Mango, Litchi and Apple but not necessarily in the same order. R has a seminar in a month which has less than 31 days. Only two people have a seminar between R and S. The one who likes Banana has a seminar immediately before T. Only one person has a seminar before the one who likes Papaya. Q has a seminar immediately after the one who likes Papaya. Only three people have a seminar between Q and the one who likes Mango. T likes neither Mango nor Papaya. P has a seminar immediately before T. V likes Apple. The one who likes Grapes has a seminar in the month, which has less than 31 days. The one who has seminar in March does not like Orange.

 

 

Q. 68 Which of the following represent the month in which S has a seminar?

A. January

B. Cannot be determined

C. October

D. December

E. June

 

Q. 69 Which of the following represent the people who have seminar in January and June respectively?

A. V, S

B. U, S

C. Q, T

D. V, R

E. U, R

 

Q. 70 How many people have a seminar between the months in which V and R have a seminar?

A. None

B. Two

C. Three

D. One

E. More than three

 

Q. 71 As per the given arrangement, R is related to Banana and P is related to orange following a certain pattern, which of the following is V related to following the same pattern?

A. Mango

B. Litchi

C. Apple

D. Papaya

E. Grapes

 

Q. 72 Which of the following fruits does U like?

A. Orange

B. Papaya

C. Mango

D. Banana

E. Grapes

 

Questions: 73 – 75

Directions (73-75) : Study the following information to answer the questions.

Each of the six buildings P, Q, R, S, T and U houses different number of offices. S has more offices than only T and R. Q has more number of offices than P but less than U. R does not house the least number of offices. The building which houses the least number of offices has 5 offices. The building which has second highest number of offices has 23 offices. S has 11 less number of offices than Q.

 

Q. 73 Which of the following buildings has the second least number of offices?

A. Q

B. U

C. R

D. P

E. T

 

Q. 74 If the number of offices in P is an even number which is divisible by 2 as well as 3. How many does P have?

A. 20

B. 24

C. 16

D. 18

E. 12

 

Q. 75 Which of the following is the number of offices in R?

A. 25

B. 12

C. 13

D. 14

E. 11

 

Questions: 76 – 80

Directions (76-80) : Study the following information to answer the given questions.

Ten people are sitting in two parallel rows containing five people each, in such a way that there is equal distance between adjacent persons. In row-1 J, K, L, M and N are seated (not necessarily in the same order) and all of them are facing south. In row-2 V, W, X, Y and Z are seated (not necessarily in the same order) and all of them are facing north. Therefore in the given seating arrangement each member seated in a row faces another member of the other row. Z sits third to the right of W. V sits second to the left of Z. The persons facing V sits to the immediate right of K. Only one person sits between K and M. J is not an immediate neighbour of K. Only two people sit between J and L. Neither K nor J faces Y.

 

Q. 76 Who among the following is facing N?

A. Y

B. Z

C. V

D. X

E. W

 

Q. 77 Which of the following statement is true regarding M?

A. M faces one of the immediate neighbours of X.

B. K is one of the immediate neighbours of M.

C. None of the given statements is true.

D. L sits to the immediate right of M.

E. Only one person sits between M and N.

 

Q. 78 Who amongst the following is facing X?

A. k

B. l

C. M

D. J

E. N

 

Q. 79 What is the position of Z with respect to Y?

A. Third to the right

B. Second to the right

C. Immediate left

D. Immediate right

E. Second to the left

 

Q. 80 Four of the given five are alike in a certain way based on the given arrangement and hence form a group. Which of them does not belong to that group?

A. M

B. J

C. N

D. W

E. Y

 

Q. 81 Two/ three statements followed by two conclusions numbered I and II have been given. You have to take the two/three given statements to be true even if they seem to be at variance from commonly known facts and then decide which of the given conclusions logically follows from the given statements disregarding commonly known facts.

Statements:

All races are sprints

Some races are contests

Conclusions:

I. Some contests are sprints

II. All contests are sprints

A. If only conclusion II is true

B. If only conclusion I is true

C. If both conclusions I and II are true

D. If either conclusion I or II is true

E. If neither conclusion I nor II is true

 

Q. 82 Two/ three statements followed by two conclusions numbered I and II have been given. You have to take the two/three given statements to be true even if they seem to be at variance from commonly known facts and then decide which of the given conclusions logically follows from the given statements disregarding commonly known facts.

Statements:

No bank is a locker

All banks are stores

No store is a panel

Conclusions:

I. No store is a locker

II. No panel is a bank

A. If only conclusion II is true

B. If only conclusion I is true

C. If both conclusions I and II are true

D. If either conclusion I or II is true

E. If neither conclusion I nor II is true

 

Q. 83 Two/ three statements followed by two conclusions numbered I and II have been given. You have to take the two/three given statements to be true even if they seem to be at variance from commonly known facts and then decide which of the given conclusions logically follows from the given statements disregarding commonly known facts.

Statements:

Some strikes are hits

No strike is a raid

All attacks are raids

Conclusions:

I. Some hits are definitely not raids

II. All hits being strikes is a possibility

A. If only conclusion II is true

B. If only conclusion I is true

C. If both conclusions I and II are true

D. If either conclusion I or II is true

E. If neither conclusion I nor II is true

 

Q. 84 Two/ three statements followed by two conclusions numbered I and II have been given. You have to take the two/three given statements to be true even if they seem to be at variance from commonly known facts and then decide which of the given conclusions logically follows from the given statements disregarding commonly known facts.

Statements:

Some equations are formulae

All equations are terms

All terms are symbols

Conclusions:

I. All equations are symbols

II. No symbol is a formulae

A. If only conclusion II is true

B. If only conclusion I is true

C. If both conclusions I and II are true

D. If either conclusion I or II is true

E. If neither conclusion I nor II is true

 

Q. 85 Two/ three statements followed by two conclusions numbered I and II have been given. You have to take the two/three given statements to be true even if they seem to be at variance from commonly known facts and then decide which of the given conclusions logically follows from the given statements disregarding commonly known facts.

Statements:

Some strikes are hits

No strike is a raid

All attacks are raids

Conclusions:

I. No attack is a strike

II. All attacks being hits is a possibility

A. If only conclusion II is true

B. If only conclusion I is true

C. If both conclusions I and II are true

D. If either conclusion I or II is true

E. If neither conclusion I nor II is true

 

Questions: 86 – 90

Directions (86-90) : study the following information to answer the given questions.

‘festival for women only’ is written as ‘pa ge bo xu’

‘provide peace to women’ is written as ‘wr dl nj ge’

‘women like to celebrate’ is written as ‘ge ct fx wr’

‘celebrate peace in festival’ is written as ‘dl bo sv ct’

(All codes are two letter codes only)

 

Q. 86 What may be possible code for ‘provide idea’ in the given code language?

A. fx by

B. xu bo

C. hy nj

D. nj xu

E. wr fx

 

Q. 87 What is the code for ‘celebrate’ in the given code language?

A. sv

B. wr

C. ct

D. dl

E. fx

 

Q. 88 In the given code language, what does the code ‘pa’ stand for?

A. Peace

B. Either ‘for’ or ‘only’

C. Either ‘women’ or ‘to’

D. celebrate

E. festival

 

Q. 89 What is the code for ‘women’ in the given code language?

A. bo

B. xu

C. ct

D. other than those given as option

E. ge

 

Q. 90 If ‘peace to mind’ is coded as ‘zg wr dl’ in the given code language, then what is the code for ‘mind in festival’?

A. zg bo dl

B. dl zg sv

C. bo sv zg

D. zg nj wr

E. sv wr bo

 

Q. 91 Relationship between different elements is shown in the statements.The statements are followed by the conclusion. Study the conclusion based on the given statements and select the appropriate answer

Statements:

S≤L≤I=P≥E>R ; L>Q

Conclusions:

(I) P ≥ S

(II) I > R

A. If only conclusion (II) is true

B. If only conclusion (I) is true

C. If both conclusions (I) and (II) are true

D. If either conclusions (I) or (II) are true

E. If neither conclusions (I) nor (II) are true

 

Q. 92 Relationship between different elements is shown in the statements.The statements are followed by the conclusion. Study the conclusion based on the given statements and select the appropriate answer

Statements :

G > R ≥ E = A ≤ T ≤ S ; D ≤ A ≤ J

Conclusions : I. T ≥ D II. R > S

A. If only conclusion II is true

B. If only conclusion I is true

C. If both conclusions I and II are true

D. If either conclusion I or II is true

E. If neither conclusion I nor II is true

 

Q. 93 Relationship between different elements is shown in the statements.The statements are followed by the conclusion. Study the conclusion based on the given statements and select the appropriate answer

Statements :

A ≥ B > C ≤ D ≤ E < F

Conclusions : I. A ≥ E II. C < F

A. If only conclusion II is true

B. If only conclusion I is true

C. If both conclusions I and II are true

D. If either conclusion I or II is true

E. If neither conclusion I nor II is true

 

Q. 94 Relationship between different elements is shown in the statements.The statements are followed by the conclusion. Study the conclusion based on the given statements and select the appropriate answer

Statements :

G > R ≥ E = A ≤ T ≤ S ; D ≤ A ≤ J

Conclusions : I. J > G II. J = G

A. If only conclusion II is true

B. If only conclusion I is true

C. If both conclusions I and II are true

D. If either conclusion I or II is true

E. If neither conclusion I nor II is true

 

Q. 95 Relationship between different elements is shown in the statements.The statements are followed by the conclusion. Study the conclusion based on the given statements and select the appropriate answer

Statements :

S ≤ L ≤ I = P ≥ E > R ; L > Q

Conclusions : I. L < R II. E ≥ Q

A. If only conclusion II is true

B. If only conclusion I is true

C. If both conclusions I and II are true

D. If either conclusion I or II is true

E. If neither conclusion I nor II is true

 

Questions: 96 – 100

Directions (96-100) : study the following information to answer the given questions.

J, K, L, M, N, O, P, Q, R, and S are sitting around a circular table facing the centre with equal distances between each other (but not necessarily in same order). Each one of them is also related to M in some way or the other. Only two people sit between Q and L. M sits second to the left of Q. Only three people sit between L and M’s sister. M’s son sits second to the right of M’s sister. Only one person sits between M’s son and S. J sits to the immediate right of R. R is neither the son nor the mother of M. S is an immediate neighbour of M’s mother. Only three people sit between M’s mother and M’s brother. M’s daughter sits second to the left of M’s brother. M’s father is not an immediate neighbour of M. M’s wife sits third to the right of K.

 

Q. 96 Who sits second to the right of R?

A. M’s brother

B. M

C. R

D. K

E. M’s daughter

 

Q. 97 How many people sit between K and L, when counted from the left of L?

A. Six

B. One

C. None

D. Two

E. Four

 

Q. 98 Which of the following statements is true with respect to the given information?

A. R sits second to the right of M’s wife

B. K is an immediate neighbour of R.

C. M sits second to the left of L.

D. All the given options are true.

E. S is the daughter of L.

 

Q. 99 How is K related to R?

A. Son-in-law

B. Uncle

C. Niece

D. Brother

E. Daughter

 

Q. 100 Who amongst the following is the wife of M?

A. N

B. L

C. O

D. Q

E. J

 

 

Answer Sheet
Question 1 2 3 4 5 6 7 8 9 10
Answer B D A B C C C A A C
Question 11 12 13 14 15 16 17 18 19 20
Answer A D D A D B A E B A
Question 21 22 23 24 25 26 27 28 29 30
Answer B E B D C E B D C B
Question 31 32 33 34 35 36 37 38 39 40
Answer E C A D C E C B C D
Question 41 42 43 44 45 46 47 48 49 50
Answer B D C B B C D E C C
Question 51 52 53 54 55 56 57 58 59 60
Answer E B D C E E A C E C
Question 61 62 63 64 65 66 67 68 69 70
Answer B E C B B D D D D B
Question 71 72 73 74 75 76 77 78 79 80
Answer D B C D E C D A C C
Question 81 82 83 84 85 86 87 88 89 90
Answer B A B B C C C B E C
Question 91 92 93 94 95 96 97 98 99 100
Answer C B A E E B C E C E

IBPS Clerk 2017 Prelims Previous Year Paper

IBPS Clerk 2017 Prelims

Section

Questions

Marks

English

30 Questions (1 – 30)

Reasoning

35 Questions

Mathematics

35 Questions

Q : 1 – 5

Direction (1 – 5) : Read the given passage carefully and answer the questions that follow.

When Hurricane Harvey loomed off the coast of my home state of Texas, it seemed to fill the entire Gulf of Mexico. When it roared on land, it pummelled the towns of Rockport and Port Aransas, whose tawny beaches I’ve walked with my kids, pointing out the indigo sails of Portuguese man o’ war jellyfish. Harvey’s eye took direct aim at the University of Texas’ Marine Science Institute, flattening not just the facility itself, but priceless samples awaiting analysis. After Harvey left Port Aransas, it spun back into the Gulf of Mexico over record sea temperatures as great as 4 degrees Fahrenheit above normal. Thermodynamic laws require that warmer air holds more water vapor. The heat armed the storm with a mighty arsenal of water vapor. Then Harvey returned to land, dumping a catastrophic amount of rain on Houston. My Facebook feed filled with pleas for rescue from the rising waters. Friends’ houses flooded —houses that had always been on dry land before. A chemical plant blew up, twice. Toxic chemicals oozed from Superfund sites. Dozens died in the deluge, mostly by drowning. And all the while, alongside the heartbreak and horror, I kept thinking about a strange harbinger: jellyfish. Diaphanous in form yet menacing in sting, jellyfish have a powerful capacity to capture our imagination. They undulate in a primal rhythm, blinking open and closed like eyes that can peer into the soul of the sea. And what they are seeing are changes produced by us here on land. Because we burn fossil fuels, which release greenhouse gasses, not just the atmosphere but ocean waters are warming. At the same time, our ship traffic transports animals to new places, and sometimes these exotics find home-like conditions where in the past those conditions would have been unsuitable. That’s likely what happened off the coast of Italy, where gas platforms are thought to be the home for a new invasion of jellyfish. In the twentieth century in the Adriatic Sea, moon jellies, pinkish with their characteristic four-leafed clover on top, were a rarity. Now they are ubiquitous. And as we wash pollution into our waters, we create low oxygen environments. Some jellyfish, with their low metabolic rate due to their a-cellular jelly insides, can survive more easily there than fish, with their oxygen-guzzling muscled tissues. The fierce stings of these animals chase beach- goers out of the water. Their gooey bodies clog machinery at power plants, halting operations. Rampant coastal development provides new habitat for a jellyfish stage called a polyp that looks like a sea anemone. When it finds a hard surface like a dock or a jetty to grow on, a single polyp can proliferate into a dozen or even more medusae. And fields of polyps grow on those hard structures. And our lack of oversight of the fishing industry, which has removed more than 90% of the large fish from the seas, has depleted the predators of jellyfish as well as their competitors. Jellyfish are eaten by some fish, and jellyfish eat the same small zooplankton that fish do. The ecological vacuum left by unrestrained fishing can allow jellyfish to expand their influence in marine ecosystems. That’s what happened off the coast of Namibia, once one of the world’s most productive fisheries.

 

Q. 1 Jellyfish can survive in polluted water better than other fishes. Why?

I. They have oxygen gulping muscled tissues.

II. They have low metabolic rate.

III. They eat zooplankton for survival.

A. Only I

B. Only II

C. Both I and II

D. Both II and III

E. All are correct

 

Q. 2 Why Hurricane Harvey is termed as devastating?

A. Many houses got flooded during the hurricane.

B. Many people died due to drowning.

C. Toxic chemicals got released from superfund sites.

D. Chemical plants got blown up.

E. All of the above

 

Q. 3 What is the main idea of the passage?

A. Jellyfish is an endangered species.

B. Low oxygen content due to water pollution has caused changes in aquatic ecosystem.

C. Informing some unknown facts about jellyfish.

D. Destruction caused by Hurricane Harvey.

E. How to improve ecological balance.

 

Q. 4 Pick up the fact(s) about Jellyfish that is(are) related with Namibia.

A. Low oxygen content has caused reduction in the number of jellyfish.

B. Jellyfish can extensively increase their population in case of ecological imbalance.

C. Unregulated fishing activities has led to a stark decrease in number of Jellyfish predators.

D. Both B and C.

E. All are correct

 

Q. 5 Which of the following statement(s) is(are) true in the context of the given passage?

A. Moon Jellyfish is found in Yellow sea where pollution is unchecked.

B. Hurricane Harvey with strong winds battered the entire Gulf of Mexico.

C. Jellyfish sweeping from China have afflicted Japan’s coast.

D. Jellyfish survive more than other fishes in low oxygen environment

E. Only B and D

 

Questions: 6 – 15

Direction (6-15): In the given question, one statement with a blank is given along with four words. Two of the given words can fit into the given blank. Five options with various combinations of these words is given. Pick up the combination of the words that fit into the blank.

 

Q. 6 The bank officials have been found ________ of taking bribes.

a) Receptive

b) Guilty

c) Probable

d) Accurate

A. a-c

B. b-c

C. a-d

D. c-d

E. a-b

 

Q. 7 The Defence Minister decided to give a briefing about the increased military _______ in the contemporary times.

a) Requirements

b) Threats

c) Assistance

d) Assumptions

A. a-d

B. b-c

C. a-b

D. b-d

E. a-c

 

Q. 8 The neighbours decided to come _______ after seeing the Christmas decoration being set up in the alley.

a) Across

b) On

c) Over

d) About

A. b-d

B. a-c

C. c-d

D. a-b

E. b-c

 

Q. 9 As the Governor _______ the officials to abolish the redundant act, the established system got disrupted.

a) Asked

b) Requested

c) Ordered

d) Inquired

A. b-c

B. a-c

C. b-d

D. a-b

E. a-d

 

Q. 10 The waiter came to our table and asked if we wanted _______ coffee, as it was closing time for the cafe.

a) Much

b) More

c) Small

d) Some

A. a-b

B. b-c

C. b-d

D. c-d

E. a-d

 

Q. 11 The country’s diplomatic history is filled with instances of _______ people coming together to counter challenges.

a) Powerful

b) Insightful

c) Approximate

d) Lethargic

A. b-d

B. a-c

C. a-b

D. b-c

E. a-d

 

Q. 12 In the latter decades of the last century, it was noted that racism was ________

acknowledged by those in power.

a) Virtually

b) Speculatively

c) Strongly

d) Hardly

A. a-b

B. c-d

C. a-c

D. b-d

E. b-c

 

Q. 13 Looking at the state of disorder, the veterans often _______ about the fading relevance of their sacrifices during the great war.

a) Stuff

b) Suit

c) Wonder

d) Think

A. a-c

B. b-d

C. a-d

D. c-d

E. b-c

 

Q. 14 The contract for the dam was being heavily contested _______ the major construction giants on the continent.

a) By

b) Among

c) Across

d) Over

A. a-b

B. b-c

C. b-d

D. a-c

E. a-d

 

Q. 15 The Election Commission decided to _______ its authority after receiving complaints of malpractices during the poll campaigns.

a) Ease

b) Use

c) Exercise

d) Ascertain

A. a-c

B. b-c

C. b-d

D. a-b

E. a-d

 

Questions: 16 – 20

Direction (16-20): The given question has a statement with an idiom in it, which has been mentioned in bold. The statement is followed by five options; pick the option that best explains the meaning of the idiom, as used in the statement.

 

Q. 16 The company was keen at cutting down on the employee overheads in order to make the business profitable.

A. Reduce

B. Spike

C. Ensure

D. Elongate

E. Affiliate

 

Q. 17 It was difficult to put up with the nagging relatives during the growth years.

A. Decorate

B. Issue

C. Endure

D. Richochet

E. Prioritise

 

Q. 18 The commissioner advised the fugitives to turn in themselves to get some legal respite.

A. Lessen

B. Accentuate

C. Retard

D. Liberate

E. Surrender

 

Q. 19 Aniket was willing to go overboard for receiving a scholarship to pursue higher education overseas.

A. Remove

B. Stretch

C. Incur

D. Liquidate

E. Enhance

 

Q. 20 The central bank was of the idea that it was not suitable to bail out the ailing cooperatives.

A. Depart

B. Alter

C. Entertain

D. Reverse

E. Rescue

 

Questions: 21 – 30

Direction (21-30): The given question contains one statement that is divided into five parts, out of which the first part is correct. There is some error in the three of the remaining four parts of the statement, while one part is correct. The correct part is your answer. In case all the parts are incorrect, the last option is your answer.

 

Q. 21 The trouble with/ mental illness are/ that it is rarely/ diagnosed latter than/ it should suitably be.

A. mental illness are

B. that it is rarely

C. diagnosed latter than

D. it should suitably be

E. All are incorrect

 

Q. 22 Recurrence of scams/ in the recent/ decades can be credited by/ the increasing in/ the bureaucratic lenient.

A. in the recent

B. decades can be credited by

C. the increasing in

D. the bureaucratic lenient.

E. All are incorrect

 

Q. 23 The proactive approach of/ a city authorities / have led to the reduction/ with unwanted federal expenses/ in tax collection.

A. a city authorities

B. have led to the reduction

C. with unwanted federal expenses

D. in tax collection.

E. All are incorrect

 

Q. 24 It has always been/ difficult with the political/machinery to meshing with/ the judiciary because/of the conflicts of interest.

A. difficult with the political

B. machinery to meshing with

C. the judiciary because

D. of the conflicts of interest

E. All are incorrect

 

Q. 25 My driver revved the engine/ hard in an attempt to increasing/ the available power over/ the engine, but failed/ due to the favorable weather.

A. hard in an attempt to increasing

B. the available power over

C. the engine, but failed

D. due to the favorable weather.

E. All are incorrect

 

Q. 26 There has been/ a unprecedented increase in/ the number of adoption/ requests into the last/ few years of collect statistics.

A. a unprecedented increase in

B. he number of adoption

C. requests into the last

D. few years of collect statistics.

E. All are incorrect

 

Q. 27 The incumbent Greek government/ is willing to make most sacrifices/ than their

predecessors in/ order to impress financial/ situation in the troubled economics.

A. is willing to make most sacrifices

B. than their predecessors in

C. order to impress financial

D. situation in the troubled economics.

E. All are incorrect

 

Q. 28 Removing the trash from/ the street was being always/ consideration a prerequisite/ for improvement the overall/ ambience of the city.

A. the street was being always

B. consideration a prerequisite

C. for improvement the overall

D. ambience of the city.

E. All are incorrect

 

Q. 29 Referring to the attention/ received for the pilot project,/ the district collector

commemoration / the social workers involved in/ the execution of an crucial plan.

A. received for the pilot project

B. the district collector commemoration

C. the social workers involved in

D. the execution of an crucial plan.

E. All are incorrect

 

Q. 30 Living under the shadow/ of the revolting must have/ has deteriorating effect in/ the personalities of the childrens/ growing over these years.

A. of the revolting must have

B. has deteriorating effect on

C. the personalities of the childrens

D. growing over these years

E. All are incorrect

 

Questions: 31 – 35

Direction: What should come in the place of question mark (?) in the following number series?

 

Q. 31 7, 10, 16, 28, ?, 100

A. 34

B. 40

C. 52

D. 60

E. 78

 

Q. 32 9, 11, 16, 26, ?, 69

A. 31

B. 38

C. 43

D. 45

E. 46

 

Q. 33 9, 11, 16, 26, ?, 69

A. 31

B. 38

C. 43

D. 45

E. 46

 

Q. 34 3, 4, 10, 33, 136, ?

A. 150

B. 298

C. 463

D. 572

E. 685

 

Q. 35 2, 3, 6, 15, 45, ?

A. 90

B. 135

C. 157.5

D. 200

E. 225

 

Questions: 36 – 40

Directions (36-40): Study the following Table and answer the questions given below.

 

Q. 36 What was the respective ratio between the Books sold in Shop P to Shop S in the year 2005?

A. 243:250

B. 343:250

C. 204:211

D. 104:115

E. None of these

 

Q. 37 What was total number of Books sold in shop R in all the years together?

A. 84200

B. 42400

C. 71900

D. 72800

E. none of these

 

Q. 38 Which Shops sold maximum number of Books in all the years together?

A. P

B. Q

C. R

D. T

E. none of these

 

Q. 39 What was the difference between the numbers of books sold in shop T in 2001 and In Shop S in the year 2003?

A. 600

B. 900

C. 400

D. 450

E. None of these

 

Q. 40 What was the total number of books sold in Shop Q in 2003, 2004, and 2005?

A. 36500

B. 38500

C. 37500

D. 39000

E. None of these

 

Q. 41 Direction: What value should come in place of the question mark (?) in the following question?

12.5×14÷8.75+42=50+?

A. 18

B. 12

C. 32

D. 42

E. 65

 

Q. 42 Directions: What value should come in place of question mark (?) in the following question? 150% of 460+24% of 650=?

A. 854

B. 854

C. 895

D. 825

E. 864

 

Q. 43 Directions: What value should come in place of question mark (?) in the following question?

(44)²+(21)²=(?)²⁺(4)⁴

A. 47

B. 65

C. 52

D. 41

E. 87

 

Q. 44 Direction: What will come in place of the question mark (?) in the following question?

504÷42×7-63+28=?

A. 72

B. 48

C. 62

D. 38

E. none of these

 

Q. 45 Direction: What value should come in place of the question mark (?) in the following question?

68 +√?-3421=591

A. 3249

B. 3481

C. 3364

D. 3136

E. none of these

 

Q. 46 Direction: What value should come in place of the question mark (?) in the following question?

(?)² % of 650 = (20)² + (4)²

A. 8

B. 64

C. √8

D. (64)²

E. 32

 

Q. 47 Direction: What value should come in place of the question mark (?) in the following question?

(5²×14+1450)/18=1998÷?

A. 18.25

B. 19.98

C. 34.58

D. 65.25

E. 45.85

 

Q. 48 Direction: What should come at the place of question mark (?) in the following question?

A. 2.79

B. 5.01

C. 4.29

D. 6.36

E. 3.96

 

Q. 49 Direction: What value should come in place of the question mark (?) in the following question?

325-(12)²+75=(?)²-68

A. √18

B. 324

C. 18

D. (324)²

E. √314

 

Q. 50 Direction: What value should come in place of the question mark (?) in the following question?

534.596 + 61.472 – 496.708 = ? + 27.271

A. 126.631

B. 62.069

C. 72.089

D. 132.788

E. none of these

 

Q. 51 Direction: What value should come in place of the question mark (?) in the following question?

√529* 6 / 2 * ?= 14076

A. 204

B. 251

C. 102

D. 146

E. 129

 

Q. 52 Direction: What value should come in place of the question mark (?) in the following question?

1637 + 1832 = (45)² + (?)²

A. 38

B. 42

C. 46

D. 54

E. 50

 

Q. 53 Direction: What value should come in place of the question mark (?) in the following question?

264 ÷√576+(11)²+12=(?)²

A. 44

B. 12

C. 15

D. 14

E. none of these

 

Q. 54 Directions: What value should come in place of question mark (?) in the following question?

69÷3×0.85+24.5-13.8=?

A. 31.75

B. 30.25

C. 28.95

D. 35.25

E. 37.95

 

Q. 55 Directions: What will come in place of question mark (?) in the following question?

A. 10550

B. 10540

C. 10580

D. 10500

E. none of these

 

Q. 56 Two items A and B have equal cost price. Item A is sold at 40% profit and item B is sold at an amount 20% less than the selling price of item A. If the total profit is 156 Rs then find the cost price of A.

A. Rs. 300

B. Rs. 350

C. Rs. 400

D. Rs. 450

E. none of these

 

Q. 57 Of three positive numbers, the ratio of the first and second numbers is 3:4 & that of the second and third numbers is 5:6. The product of the second and third numbers is 4320. What is the sum of three numbers?

A. 167

B. 177

C. 187

D. 197

E. none of these

Q. 58 The ratio of the speed of a boat downstream and speed of the stream is 9:1. If the speed of the current is 3 km per hr, find the distance travelled by the boat upstream in 5 hours.

A. 100 km

B. 98 km

C. 109 km

D. 105 km

E. 112 km

 

Q. 59 The difference between the simple interest obtained by investing Rs. ‘X’ with 8% per annum for an year and by investing Rs. ‘X+1400’ with 8% per annum for two years is 240 Rs. Find the value of X.

A. 200

B. 100

C. 400

D. 300

E. none of these

 

Q. 60 Average age of A and B, 2 years ago was 26. If the age of A, 5 years hence is 40 years, and B is 5 years younger to C, then find the difference between the age of A and C ?

A. 6

B. 9

C. 12

D. 15

E. 18

 

Q. 61 The average of X, Y, Z is 24. Also, X:Y = 2:3 and X+Y = 60, then find the value of (X-Z)

A. 9

B. 10

C. 11

D. 12

E. none of these

 

Q. 62 The length of a rectangle is 80% of the diagonal of a square of area 1225 cm², then find area of rectangle if it’s perimeter is 94√2 cm

A. 1064 cm²

B. 1077 cm²

C. 1081 cm²

D. 1090 cm²

E. none of these

 

Q. 63 The annual salary of Arun is 7.68 lac. If he spends Rs. 12000 on his children, 1/13th of rest of the salary on food and Rs. 8000 in mutual funds, then find the monthly saving he is left with.

A. Rs. 38000/-

B. Rs. 39500/-

C. Rs. 40000/-

D. Rs. 41250/-

E. Rs. 45000/-

 

Q. 64 A can do a work in 24 days. B is 20% more efficient than A. If C can do the work in 10 more days than B, find the time taken by A and C together to complete the work.

A. 43/3 days

B. 40/3 days

C. 50/3 days

D. 41/2 days

E. none of these

 

Q. 65 The ratio of milk to water is 5:4. If two litres of water is added, the ratio becomes 10:9, then find the new amount of water in the mixture.

A. 12 litres

B. 14 litres

C. 16 litres

D. 17 litres

E. 18 litres

 

Questions: 66 – 70

Direction (66-70): Study the following information carefully and answer the questions.

A building has eight floors numbered one to eight, in such a manner that the ground floor is numbered one, the floor above it, numbered two and so on such that the topmost floor is numbered eight. One of the eight persons, viz, P, Q, R, S, T, U, V and W lives on each floor. R lives on third numbered floor. Only two persons live between the floors of R and V. W lives on the floor immediately above the floor of Q. Only one person lives between the floors of T and U. T lives above U. Only one person lives between the floors of R and S. S lives on any floor below the floor of T.

 

Q. 66 Who among the following lives on the fifth numbered floor?

A. S

B. Q

C. W

D. P

E. V

 

Q. 67 Who among the following lives exactly between the floors of R and S?

A. U

B. P

C. V

D. T

E. W

 

Q. 68 Who among the following lives on the topmost floor?

A. T

B. Q

C. W

D. R

E. V

 

Q. 69 Four of the following five are alike in a certain way and hence they form a group. Which one of the following does not belong to that group?

A. R

B. V

C. P

D. S

E. Q

 

Q. 70 How many persons live between the floors of Q and U?

A. None

B. One

C. Two

D. Three

E. Four

 

Questions: 71 – 75

Direction (71-75): The question has two statements followed by two conclusions numbered I and II. You have to take the two given statements to be true even if they seem to be at variance from commonly known facts and then decide which of the given conclusions logically follows from the given statements disregarding commonly known facts.

 

Q. 71 Statements:

All buses are car.

Some buses are trucks.

Conclusions:

I. Some buses are definitely not trucks.

II. At least some trucks are cars.

A. Only conclusion I follows.

B. Only conclusion II follows.

C. Either conclusion I or II follows.

D. Neither conclusion I nor II follows.

E. Both conclusions I and II follow.

 

Q. 72 Statements:

Some animals are plants.

All plants are rocks.

Conclusions:

I. All plants are animals.

II. At least some rocks are animals.

A. Only Conclusion I follows

B. Only Conclusion II follows

C. Either Conclusion I or II follows

D. Neither Conclusion I or II follows

E. Both Conclusion I and II follow

 

Q. 73 Statement:

No pin is a clip.

All pins are fans.

Conclusions:

I. All fans are pins.

II. No clip is a fan.

A. Only conclusion I follows

B. Only conclusion II follows

C. Either I or II follows

D. Neither I nor II follows

E. Both I and II follows.

 

Q. 74 Statements:

Some kites are birds.

No kite is an aeroplane.

Conclusions :

I. All aeroplanes are birds.

II. Some birds are definitely not kites.

A. Only conclusion I follows

B. Only conclusion II follows

C. Either I or II follows

D. Neither I nor II follows.

E. Both I and II follows.

 

Q. 75 Statements:

All metals are plastics.

All plastics are fibres.

Conclusions:

I. At least some fibres are metals.

II. Some metals are not fibres.

A. Only conclusion I follows

B. Only conclusion II follows

C. Either I or II follows

D. Neither I nor II follows.

E. Both I and II follows.

 

Questions: 76 – 80

Direction (76-80): Study the following arrangement of numbers, letters, and symbols carefully and answer the questions.

I 4 M 6 % 1 R O # 9 @ G W S 2 U $ *8 7 C E 3 © K 5

 

Q. 76 How many such numbers are there in the above arrangement each of which is immediately followed by a vowel but not immediately preceded by a number?

A. None

B. One

C. Two

D. Three

E. more than three

 

Q. 77 Four of the following five are alike in a certain way on the basis of their positions in the above arrangement and so form a group. Which is the one that does not belong to that group?

A. M1%

B. U8*

C. G2S

D. 35K

E. C©E

 

Q. 78 Which of the following is seventh to the right of fifteenth from the right end?

A. 8

B. G

C. 7

D. C

E. None of these

 

Q. 79 How many such symbols are there in the above arrangement, each of which is immediately followed by a consonant and also immediately preceded by a number?

A. None

B. One

C. Two

D. Three

E. More than three

 

Q. 80 If all the symbols are dropped from the above arrangement which of the following will be eight from the left end?

A. G

B. R

C. O

D. 9

E. None of these

 

Questions: 81 – 85

Direction (81-85): In these questions, relationship between different elements is shown in the statements. These statements are followed by two conclusions.

 

Q. 81 Statement: P ≤ Q = R > S > T

Conclusions:

I. P < T

II. T < Q

A. Only conclusion I follows

B. Only conclusion II follows

C. Either conclusion I or II follows

D. Neither conclusion I nor II follows

E. Both conclusion I and II follows

 

Q. 82 Statement: L ≤ M < N > O = P

Conclusion:

I. P < N

II. O < M

A. Only conclusion I follows

B. Only conclusion II follows

C. Either conclusion I or II follows

D. Neither conclusion I nor II follows

E. Both conclusion I and II follows

 

Q. 83 Statement: J > K ≤ L = M < N

Conclusions:

I. K < N

II. K < M

A. Only conclusion I follows

B. Only conclusion II follows

C. Either conclusion I or II follows

D. Neither conclusion I nor II follows

E. Both conclusion I and II follows

 

Q. 84 Statement: P ≤Q = R, T > R = S

Conclusions:

I. P = S

II. P < S

A. Only conclusion I follows

B. Only conclusion II follows

C. Either conclusion I or II follows

D. Neither conclusion I nor II follows

E. Both conclusion I and II follows

 

Q. 85 Statement: P ≤ Q = R, T > R = S

Conclusions:

I. Q < T

II. P < S

A. Only conclusion I follows

B. Only conclusion II follows

C. Either conclusion I or II follows

D. Neither conclusion I nor II follows

E. Both conclusion I and II follows

 

Questions: 86 – 90

Direction (86-90): Study the information given below and answer the question based on it.

Eight persons A, B, C, D, E, F, G and H are sitting around a circular table and facing the centre. They earn different salaries i.e. 12000, 15000, 17000, 18000, 21000, 22000, 26000 and 32000, but not necessarily in the same order. D sits third to the left of A who earns 12000. The one who earns 18000 is neighbor of A. F earns 4000 more than G. Two persons sit between the one who earns 18000 and H. B earns 17000 and sits immediate left of D. G and C are neighbours and earn total 33000. F is neighbor of A and the one who earns the highest. D earns more than E. The one who earns 21000 sits 2 nd to the right of the one who earns 18000.

 

Q. 86 Who among the following earns the highest?

A. F

B. C

C. H

D. G

E. D

 

Q. 87 Which of the following combination is correct?

A. D-21000

B. C-18000

C. G-18000

D. G-15000

E. F-21000

 

Q. 88 Which of the following pair is sitting opposite?

A. B and C

B. C and H

C. A and E

D. E and H

E. G and B

 

Q. 89 Who is sitting exactly between G and E?

A. The one who earns 18000.

B. A

C. F

D. The one who earns 15000

E. B

 

Q. 90 How many persons are earning more than D?

A. none

B. 1

C. 2

D. 3

E. 4

 

Questions: 91 – 95

Directions (91-95): Study the information given below and answer the questions based on it.

OTM NSP AKG BCE LTR

 

Q. 91 If all the alphabets in each word are arranged in reverse English alphabetical order then which of the following word comes first in reverse English alphabetical order?

A. OTM

B. NSP

C. AKG

D. BCE

E. LTR

 

Q. 92 If in each of the words, the first alphabet is changed to previous alphabet according to English alphabetical series and last alphabet is changed to the next alphabet according to English alphabetical series so in how many words there is no vowel in that particular word?

A. 0

B. 1

C. 2

D. 3

E. 4

 

Q. 93 If the second letter is replaced with an alphabet which succeeds the second alphabet is English alphabetical order and the last letter is replaced with an alphabet which precedes the last letter in English alphabetical order then how many words will have repeated alphabets?

A. 0

B. 1

C. 2

D. 3

E. 4

 

Q. 94 If in each of the words, we sum all the alphabets number according to their position in English alphabetical series (i.e. A=1) so which of the following get the highest number?

A. OTM

B. NSP

C. AKG

D. BCE

E. LTR

 

Q. 95 If in each of the words, all the consonants are replaced with an alphabet which succeeds that consonant in English alphabetical order and all the vowels are replaced with an alphabet which precedes that vowel in English alphabetical order then how many words will be there with at least two vowels?

A. 0

B. 1

C. 2

D. 3

E. 4

 

Questions: 96 – 100

Direction (96-100): Study the following information carefully and answer the questions.

Eight persons M, N, O, P, Q, R, S and T are sitting in a straight line. Four of them are facing north and four of them are facing south. Only three persons sit between M and N and N is sitting at one of the ends. O is 2 nd to the right of M and facing north. Q is 3 rd to the right of O. More than two persons sit between Q and R. T is 2 nd to the left of R. P is 2 nd to the right of Q. More than two persons sit between T and N. M’s neighbours are facing same direction. Q’s neighbours are facing same direction. P’s neighbours are facing opposite directions.

 

Q. 96 Who among the following sits at the corner?

A. R

B. Q

C. S

D. T

E. P

 

Q. 97 Who among the following is not belongs to the group?

A. R

B. M

C. S

D. Q

E. P

 

Q. 98 How many persons sit between Q and R?

A. None

B. 1

C. 2

D. 3

E. 4

 

Q. 99 Who among the following is 3 rd to the left of S?

A. No one

B. M

C. Q

D. T

E. R

 

Q. 100 Who among the following exactly between M and O?

A. T

B. R

C. P

D. N

E. Q

 

 

Answer Sheet  
Question 1 2 3 4 5 6 7 8 9 10
Answer D E B C E B C C B C
Question 11 12 13 14 15 16 17 18 19 20
Answer C B D A B A C E B E
Question 21 22 23 24 25 26 27 28 29 30
Answer D A D C C B B D C D
Question 31 32 33 34 35 36 37 38 39 40
Answer B C C E C B C A C C
Question 41 42 43 44 45 46 47 48 49 50
Answer B B A E B A B B C C
Question 51 52 53 54 55 56 57 58 59 60
Answer A A B B C A B D A B
Question 61 62 63 64 65 66 67 68 69 70
Answer D A C B E D A C B E
Question 71 72 73 74 75 76 77 78 79 80
Answer B B D D A B E A C D
Question 81 82 83 84 85 86 87 88 89 90
Answer B A A C A C C B D B
Question 91 92 93 94 95 96 97 98 99 100
Answer E E B E A A D E A A

IBPS Clerk 2016 Prelims Previous Year Paper

IBPS Clerk 2016 Prelims Paper 1

 

Section

Quantitative Aptitude

Questions

35 Questions (1 – 35)

Marks

35

English

30 Questions (36 – 65)

30

Logical Reasoning

35 Questions (66 – 100)

35

Q : 1 – 5

What value should come in place of question-mark (?) in the following number series?

 

Q. 1 78, 294, 419, 483, ?

A. 518

B. 505

C. 515

D. 510

E. 516

 

Q. 2 16, 26, 6, 46, -34, ?

A. 126

B. 116

C. 124

D. 118

E. 125

 

Q. 3 174, 232, 298, 372, ?

A. 544

B. 464

C. 454

D. 474

E. 445

 

Q. 4 17, 35, 109, 445, ?

A. 2214

B. 2231

C. 2411

D. 2421

E. 2241

 

Q. 5 1498, 1473, 1437, 1388, ?

A. 1342

B. 1324

C. 1314

D. 1332

E. 1334

 

Q. 6 What value should come in place of the question mark (?) in the following questions ?

99 × 21 – cuberoot(?) = 1968

A. 1367631

B. 111

C. 366731

D. 1367

E. None of these

 

Q. 7 What value should come in place of the question mark (?) in the following questions ?

572 + 38 × 0.50 – 16 = ?

A. 289

B. 305

C. 448

D. 565

E. 575

 

Q. 8 What value should come in place of the question mark (?) in the following questions ?

 

6269 + 0.75 × 444 + 0.8 × 185 = ? × 15

A. 448

B. 450

C. 452

D. 454

E. 455

 

Q. 9 What value should come in place of the question mark (?) in the following questions ?

(78.95) ² – (43.35) ² = ?

A. 4353.88

B. 4305

C. 4235.78

D. 4148

E. 4100

 

Q. 10 What value should come in place of the question mark (?) in the following questions ?

1/8 of 2/3 of 3/5 of 1715 =?

A. 80

B. 85

C. 90

D. 95

E. 75

 

Q. 11 What value should come in place of the question mark (?) in the following questions ?

1190 ÷ √7225 × ? = 3094

A. 221

B. 121

C. 214

D. 241

E. None of these

 

Q. 12 What value should come in place of the question mark (?) in the following questions ?

(√5–1)² = ? – 2√5

A. 6

B. 6+2√5

C. 6√5

D. 6-2√5

E. None of these

 

Q. 13 A man has Rs. 480 in the denominations of one-rupee notes, five-rupee notes and tenrupee notes. The number of notes of each denomination is equal. What is the total number of notes that he has ?

A. 45

B. 60

C. 75

D. 90

E. None of these

 

Q. 14 There are two examinations rooms A and B. If 10 students are sent from A to B, then the number of students in each room is the same. If 20 candidates are sent from B to A, then the number of students in A is double the number of students in B. The number of students in room A is:

A. 20

B. 80

C. 100

D. 200

E. None of these

 

Q. 15 Find the value of

√(4 + √(44 + √1000))

A. 44

B. 12

C. 4

D. 2

E. none of these

 

Q. 16 √(-√(3 + √(3 + 8√(7 + 4√3))))

A. 1

B. 2

C. 3

D. 8

E. none of these

 

Q. 17 √(2√(2√(2√(2√(2))))

A. 0

B. √2

C. 2

D. 2 31/32

E. none of these

 

Q. 18 √((0.85 × (0.105 + 0.024 – 0.008)) / (0.22 × 0.25 ×17))

A. √11

B. √0.011

C. 11

D. √0.11

 

Q. 19 if x/2y = 3/2 then the value of 2x+2y/x-2y equals

A. 1/7

B. 7

C. 7.1

D. 6.8

E. None of these

 

Q. 20 If a * b = a^2 + a x b + b^2, then what is 2 * 3?

A. 29

B. 19

C. 9

D. 6

E. None of these

 

Q. 21 What is the total number of male employees taking all the banks together?

A. Other than the given options

B. 4060

C. 4120

D. 4180

E. 4280

 

Q. 22 What is the average number of female employees taking all the banks together?

A. 656

B. 686

C. 668

D. Other than the given options

E. 646

 

Q. 23 Approximately by what percent is the number of male employees working in banks A and C together more than that of the total number of female employees working in bank B and D?

A. Other than the given options

B. 9%

C. 15%

D. 11%

E. 13%

 

Q. 24 What is the ratio of female employees working in bank D to that in E?

A. 7:4

B. Other than the given options

C. 8:5

D. 7:3

E. 9:5

 

Q. 25 Approximately by what per cent is the number of total employees of bank C more than that of bank D?

A. 8%

B. 6%

C. Other than the given options

D. 4%

E. 10%

 

Q. 26 P and Q can complete a job in 24 days working together. P alone can complete it in 32 days. Both of them worked together for 8 days and then P left. The number of days Q will talk to complete the remaining work is

A. 62

B. 60

C. 65

D. 64

E. 66

 

Q. 27 A passenger sitting in a train of a certain length which is running at a speed of 60 km/hr passing through two bridges. the notices that be crossed the first bridge and second bridge in time intervals which are in the ratio of 7:4 respectively. If the length of the first bridge is 350m, then the length of the second bridge is

A. 210m

B. 220m

C. 200m

D. 190m

E. 180m

 

Q. 28 The average salary per head of all workers of an institution is Rs. 80. The average salary per head of 16 officers is Rs. 600. The average salary per head of 16 officers is Rs. 600. The average salary per head of the rest is Rs. 67. Then, the total number of workers in the institution is

A. 656

B. 665

C. 655

D. 676

E. 666

 

Q. 29 Anand finds that due to a fall in the rate of interest from 10% to 5%, his yearly income diminishes by Rs.500. His capital is ?

A. 10000

B. 12000

C. 15000

D. 18000

E. 25000

 

Q. 30 In a town, each of the 60% of families has a laptop, each of the 30% of families has a computer and each of the 15% of families has both a laptop and a computer. In all, there are 10800 families in the town. How many families in the town do not have a laptop or a computer?

A. 2600

B. 2850

C. 2900

D. 2650

E. 2700

 

Q. 31 An article of cost prince Rs. 16000 is marked at Rs. 19200. After allowing a discount of x% a profit of 15% is made. The value of x is.

A. 3.17%

B. 4.17%

C. 5.17%

D. 2.75%

E. 6.25%

 

Q. 32 464.83 + 40.14 / 7.84 + 6.26 / 0.91 = ? . Find approximate value ?

A. 375

B. 475

C. 473

D. 450

E. Other than the given options

 

Q. 33 P,Q and R started a business by investing Rs. 40,500, Rs. 45000 and Rs. 50000 respectively. After 6 months R withdrew Rs. 10,000 while P invested Rs. 4500 more. In annual profit of Rs. 53,100 the share of R will exceed that of P by

A. Rs. 1000

B. Rs. 850

C. Rs. 950

D. Rs. 800

E. Rs. 900

 

Q. 34 Three taps X,Y and Z can fill a tank in 12 hrs, 15 hrs and 20 hrs respectively. If X is open all the time and Y and Z are open for one hour each alternately, starting with Y, then the tank will be full in how many hours?

A. 10 hr

B. 6 hr

C. 8 hr

D. 7 hr

E. 9 hr

 

Q. 35 If the simple interest on a certain sum of money for 9 months at 6% is Rs. 406 less than the simple interest on the same sum for 16 months at 7% pa, then find the sum.

A. Rs 8600

B. Rs 8400

C. Rs 8800

D. Rs 8900

E. Rs 9000

 

Questions: 36 – 45

India’s tourism industry is experiencing a strong period of growth, driven by the burgeoning Indian middle class, growth in high spending foreign tourists, and coordinated government campaigns to promote ‘Incredible India’. The tourism industry in India is substantial and vibrant, and the country is fast becoming a major global destination. India’s travel and tourism industry is one of them most profitable industries in the country, and also credited with contributing a substantial amount of foreign exchange. This is illustrated by the fact that during 2006, four million tourists visited India and spent US $8.9 billion. Several reasons are cited for the growth and prosperity of India’s travel and tourism industry. Economic growth has added millions annually to the ranks of India’s middle class, a group that is driving domestic tourism growth. Disposable income in India has grown by 10.11% annually from 2001-2006, and much of that is being spent on travel. Thanks in part to its booming IT and outsourcing industry a growing number of business trips are made by foreigners to India, who will often add a weekend break or longer holiday to their trip. Foreign tourists spend more in India than almost any other country worldwide. Tourist arrivals are projected to increase by over 22% per year through till 2010, with a 33% increase in foreign exchange earnings recorded in 2004. The Tourism Ministry has also played an important role in the development of the industry, initiating advertising campaigns such as the ‘Incredible India’ campaign, which promoted India’s culture and tourist attractions in a fresh and memorable way. The campaign helped create a colorful image of India in the minds of consumers all over the world, and has directly led to an increase in the interest among tourists. The tourism industry has helped growth in other sectors as diverse as horticulture, handicrafts, agriculture, construction and even poultry. Both directly and indirectly, increased tourism in India has created jobs in a variety of related sectors. The numbers tell the story: almost 20 million people are now working in the India’s tourism industry. India’s governmental bodies have also made a significant impact in tourism by requiring that each and every state of India have a corporation to administer support issues related to tourism. A new growth sector is medical tourism. It is currently growing at around 30% per annum. Medical tourist arrivals are expected to reach one million soon. Medical tourism in Asia has grown rapidly. Medical tourism is approaching fever pitch at the tune of $4 billion US, fuelled largely by the cosmetic surgery market. One of the problems India has, despite having some world-class hospitals, is sanitation. The tourism industry of India is based on certain core nationalistic ideals and standards which are: Swaagat or welcome, Sahyog or cooperation, Soochanaa or information, Sanrachanaa or infrastructure, Suvidha or facilitation, Safaai or cleanliness and Surakshaa or security.

 

Q. 36 Choose the word which is MOST OPPOSITE in meaning of the word burgeoning as used in the passage?

A. thriving

B. sprouting

C. incorporating

D. decreasing

E. growing

 

Q. 37 Find the correct statement on the basis of the given passage.

(A) The tourism industry in our country manages to attract the world tourist.

(B) India has continued to represent a place much sought after for its diversity.

(C) India has potential to emerge as one of the world’s tourist hot spots.

A. Only (A)

B. Only (C)

C. Both (A) and (B)

D. All (A), (B) and (C)

E. Other than the given option

 

Q. 38 Choose the word which is most similar in meaning to the word “cited” as used in the passage?

A. pointed out

B. withheld

C. inserted

D. imputed

E. sourced

 

Q. 39 Which of the following statements correctly represent (s) the reason behind the expansion of India’s travel and tourism Industry?

A. India is a vast country of great beauty and diversity

B. India’s vast cultural heritage

C. The successful launch of very penetrative ad campaign

D. Business trips to India

E. All of the above

 

Q. 40 What is the meaning of the phrase “fever pitch” as used in the passage?

A. A high degree of excitement

B. A state of suffering

C. Beyond the expectations

D. Meeting the target

E. Other than the given options

 

Q. 41 Which of the following is false in respect of India’s medical tourism?

A. India’s medical tourism sector is expected to experience an annual growth rate of 30%.

B. An estimated one million tourists will travel to India for healthcare procedures every year.

C. The advantage of medical treatment in India includes reduced costs not the availability of latest medical technologies.

D. All 1), 2) and 3)

E. Other than given options

 

Q. 42 Give a suitable title to the given passage.

A. Medical tourism in India

B. Tourism Industry–A special focus of India

C. Foreign tourist arrival and foreign earning in India

D. Domestic Tourism in India

E. Employment Opportunities in Tourism

 

Q. 43 The author in the given passage is

A) skeptical about the growth of India’s medical tourism

B) of the view that lots of efforts can bring about significant changes in India’s tour and travel industry.

C) not in favor of investing on various ads

A. Only A

B. Only B

C. Only C

D. Both A and C

E. Both B and C

 

Q. 44 As mentioned in this passage, in order to developing the tourism industry the role of the tourism ministry is

A. Considerable

B. Imperceptible

C. Unique

D. Ambiguous

E. Other than the given options

 

Q. 45 Which of the following statement is true about “Incredible India” campaign?

A. It is a window policy to boost the direct as well as indirect investment in tourism.

B. It promotes India as a tourism destination

C. It pledges for best services in Air India and in Indian Rail.

D. Only 1) and 2)

E. All 1), 2) and 3)

 

Questions: 46 – 55

Russia may not be the first country that comes (1) when thoughts turn to international studies. But the truth is that this oft-underestimated, increasingly innovative country boast a long list of amazing offerings for students from all over the globe. Wondering what’s waiting for you there? Let’s count down five reasons to include Russia on your list of down five reasons to include Russia on your list of (2) international study destinations. As the world’s largest country, it’s hardly (3) that Russia is home to so many universities — 950 of them, to be exact. What may come as a surprise? How many of its higher education institutions offer world-class, globally (4) educational opportunities. Russia now participates in the Bologna Process, and many of its premier universities are members of the European Universities Association. A whopping 22 Russian universities, meanwhile, earned spots on the QS World University Rankings 2016-2017. Russia recently earned a plum spot (5) Bloomberg’s roundup of the world’s most innovative economies, but its innovative spirit is not limited to the financial sector. A Russian education uniquely fuses traditional academic (6) with a commitment to innovation in the form of competency-based education across a breadth and depth of areas of study. And while we’d be lying if we didn’t say that studying in Russia was demanding, a degree (7) a Russian institution is highly (8) by the world’s employers — making it well worth the effort. There’s a reason why Russia’s best and brightest high school grads fight so (9) for sought-after spots at the country’s universities. From engineering and economics to law and medicine, Russia offers near-endless opportunities for academic (10).

 

Q. 46 Fill in the blank (1) choosing the word that is most appropriate in the context of the passage.

A. in

B. to

C. up

D. out

 

Q. 47 Fill in the blank (2) choosing the word that is most appropriate in the context of the passage.

A. conjuncture

B. specific

C. probable

D. prospective

 

Q. 48 Fill in the blank (3) choosing the word that is most appropriate in the context of the passage.

A. surprising

B. confusing

C. believable

D. ignoring

 

Q. 49 Fill in the blank (4) choosing the word that is most appropriate in the context of the passage.

A. admitted

B. accepted

C. famed

D. recognized

 

Q. 50 Fill in the blank (5) choosing the word that is most appropriate in the context of the passage.

A. in

B. at

C. on

D. out

 

Q. 51 Fill in the blank (6) choosing the word that is most appropriate in the context of the passage.

A. destitution

B. indigence

C. privation

D. rigours

 

Q. 52 Fill in the blank (7) choosing the word that is most appropriate in the context of the passage.

A. in

B. by

C. from

D. of

 

Q. 53 Fill in the blank (8) choosing the word that is most appropriate in the context of the passage.

A. prized

B. demanding

C. authorized

D. gained

 

Q. 54 Fill in the blank (9) choosing the word that is most appropriate in the context of the passage.

A. extremely

B. vicious

C. fiercely

D. flawlessly

 

Q. 55 Fill in the blank (10) choosing the word that is most appropriate in the context of the passage.

A. amendment

B. enrichment

C. melioration

D. exaltation

 

Questions: 56 – 65

Read each sentence to find out whether there is any grammatical error in it. The error, if any, will be in one part of the sentence. The number of that part is the answer. If there is no error, the answer is (e), i.e no error. (Ignore the errors of punctuation, if any.)

 

Q. 56 We shall be calling a meeting (a)/ next week to assess (b)/ the causes of frequently (c)/ delays in infrastructure projects. (d)/ No error (e)

A. We shall be calling a meeting

B. next week to assess

C. the causes of frequently

D. delays in infrastructure projects.

E. No error

 

Q. 57 The committee has made (a)/ significant changes (b)/ in the rules which(c)/ will help to banks. (d)/ No error (e)

A. The committee has made

B. significant changes

C. in the rules which

D. will help to banks

E. No error

 

Q. 58 Graduates have experience (a)/ in the IT industry (b)/are in great demand (c)/ in the finance sector. (d)/No error (e)

A. Graduates have experience

B. in the IT industry

C. are in great demand

D. in the finance sector

E. No error

 

Q. 59 Although it has received (a)/ clearance from RBI the bank(b)/ has decided not to open(c)/their office in Malaysia. (d)/ No error (e)

A. Although it has received

B. clearance from RBI the bank(

C. has decided not to open

D. Their office in Malaysia

E. No error

 

Q. 60 English is derived from the Anglo-Saxon, (a)/ a West Germanic language, (b)/ although it’s current vocabulary includes (c)/ words from many languages. (d)/ No error. (e)

A. English is derived from the Anglo-Saxon

B. a West Germanic language

C. although it’s current vocabulary includes

D. words from many languages

E. No error

 

Q. 61 Helen Keller has proved (a)/ how language could (b)/ liberate the (c)/ blind and the deaf. (d)/ No error (e)

A. Helen Keller has proved

B. how language could

C. liberate the

D. blind and the deaf

E. No error

 

Q. 62 Half of the term have passed and yet I have not done much reading. (a)/ I am afraid that I may not (b)/ catch up with others. (c)/ What is worse? I may fail. (d)/ No error (e)

A. Half of the term have passed and yet I have not done much reading

B. I am afraid that I may not

C. catch up with others

D. What is worse? I may fail

E. No error

 

Q. 63 Aggression in some teenage boys (a)/ may be linkage to overly (b)/ large glands in their brains, (c)/ a new study has found. (d)/ No error (e)

A. Aggression in some teenage boys

B. may be linkage to overly

C. large glands in their brains,

D. a new study has found

E. No error

 

Q. 64 The tennis player easy through (a)/ the opening set before her opponent, (b)/ rallied to take the final two sets (c) / for the biggest victory of her young career. (d)/ No error (e)

A. The tennis player easy through

B. the opening set before her opponent,

C. rallied to take the final two sets

D. for the biggest victory of her young career.

E. No error

 

Q. 65 In response to the growing crisis, (a)/ the agency is urgently asking for (b)/ more contributions, to make up for (c)/ its sharp decline in purchasing power. (d)/ No error (e)

A. In response to the growing crisis,

B. the agency is urgently asking for

C. more contributions, to make up for

D. its sharp decline in purchasing power

E. No error

 

Questions: 66 – 70

Following questions are based on given information below and you’ve to tell which conclusion(s) follow:

 

Q. 66 Statements: P< A ≤ C ≥ K; C ≤ O < L; R ≤ N ≤ C

Conclusions: I . A = K II. R < L

A. Only Conclusion I is true.

B. Only Conclusion II is true.

C. Either Conclusion I or II is true

D. Neither conclusion I nor II is true

E. Both conclusions I and II are true

 

Q. 67 Statements: S ≤ T ≤ K ≤ E; N ≥ E ≤ P

Conclusions: I. S ≤ N II. S ≤ P

A. Only Conclusion I is true

B. Only Conclusion II is true

C. Either Conclusion I or II is true

D. Neither conclusion I nor II is true

E. Both conclusions I and II are true

 

Q. 68 Statements: S > O ≥ U ≥ P; O ≤ T < R; T ≥ E > A

Conclusions: I. R < A II. U = A

A. Only Conclusion I is true

B. Only Conclusion II is true

C. Either Conclusion I or II is true

D. Neither conclusion I nor II is true

E. Both conclusions I and II are true

 

Q. 69 Statements: P< A ≤ C ≥ K; C ≤ O < L; R ≤ N ≤ C

Conclusions: I. O ≥ R II. P ≤ O

A. Only Conclusion I is true

B. Only Conclusion II is true

C. Either Conclusion I or II is true

D. Neither conclusion I nor II is true

E. Both conclusions I and II are true

 

Q. 70 Statements: S > O ≥ U ≥ P; O ≤ T < R; T ≥ E > A

Conclusions: I. T > P II. T = P

A. Only Conclusion I is true

B. Only Conclusion II is true

C. Either Conclusion I or II is true

D. Neither conclusion I nor II is true

E. Both conclusions I and II are true

 

Questions: 71 – 75

Study the following series of alpha-numeric-symbol combination and answer the questions that follow:

 

Q. 71 S K 6 £ Q 2 R * C F 8 E $ G 2 # 4 9 L N 3 U V 5 Y a B 7 W 9

How many symbols are there in the above arrangement each of which is immediately preceded by but not immediately followed by a letter of English alphabet?

A. Nil

B. One

C. Two

D. Three

E. None of these

 

Q. 72 What should come in place of question mark in the following series?

S 9 K 6 W £ Q 7 2 ? C a F

A. R Y *

B. *Y C

C. 2 B R

D. R B *

E. None of these

 

Q. 73 If the positions of # and $ are interchanged, so also the positions of £ and ê,Q and K and F and V are interchanged, which of the following will be the ninth element to the left of eighteenth element from the left?

A. C

B. V

C. 8

D. *

E. None of these

 

Q. 74 Four of the following five are alike in respect of their positions in the above series. Which is the one that does not belong to that group?

A. Q * 8

B. £ R F

C. 6 2 C

D. 2 * 8

E. R F $

 

Q. 75 S K £ is related to 9 W B in the same way as Q 2 * is related to

A. a V U

B. a Y 5

C. 7 B Y

D. B Y V

E. a Y V

 

Questions: 76 – 81

Study the following information carefully and answer the questions given below:

There are eight persons viz, A, B, C, D, E, F, H and J are sitting around a circle. Five of them are facing towards the centre. They read different newspapers viz, Hindustan Times (H.T), The Indian Express, The Hindu, The Economic Times, Navbharat Times, Washington Post, The Herald and The Dawn, but not necessarily in the same order. D is facing away from the centre but reads neither “The Hindus ‘nor’ The Indian Express.’ The one who reads The Washington post sits exactly between F and B. C is on the immediate right of H, who is not facing away from the centre. The one who reads The Indian Express is not the neighbour of B. The one who reads The Economic Times is on the immediate right of C. There are three persons between H and B. E sits second to the left of D and he reads The Indian Express. F is an immediate neighbour of both C and the one who reads The Washington Post. The Economics Times is read by either H or B. The one who read ‘The Herald’ is facing towards the centre. D sits on the immediate right of the one who read Navbharat Times. The one who reads The Dawn sits exactly between J and the one who reads The Hindu. A does not read The Washington Post. The one who is on the immediate right of H read the Hindu and is not facing the centre.

 

Q. 76 Which of the following papers is read by the one who sits on the immediate right of J?

A. The Hindu

B. The Herald

C. The Dawn

D. Navbharat Times

E. The Economic Times

 

Q. 77 Which of the following is matched correctly?

A. A- Hindustan Times

B. H- The Dawn

C. C- The Economic Times

D. B- The Herald

E. E- The Hindu

 

Q. 78 Which of the following groups is facing away from the centre?

A. A, B, D

B. A, C, D

C. E, A, D

D. J, C, D

E. H, D, B

 

Q. 79 If D is related to A and in the same way H is related to C in a certain way, then F is related to

A. J

B. C

C. None

D. A

E. B

 

Q. 80 How many persons sit between the one who reads The Dawn and the one who reads Navbharat Times?

A. Three

B. Two

C. One

D. None

E. Other than the given options

 

Q. 81 Who among the following reads The Dawn?

A. A

B. H

C. F

D. B

E. E

 

Questions: 82 – 86

Study the following information carefully and answer the questions given below:

Eight person K,C,V,D,M,O,T and L are sitting around a square table, The Persons sitting at the corner are facing the center and the persons sitting in the middle are facing away from the centre. Each one of them like a different subjects away from the centre. Each one of them like a different subjects viz, English, Hindi, Civics, Geography, Physics, Biology, History and Chemistry but not necessary in the name order.

V is facing outward and sits third to the left of C, who likes Geography.

K is not facing towards the table and he sits between the persons who like Civics and Hindi.

sits opposite K and likes English

C and L are opposite each other. their favorite subjects are geography and

Civics respectively.

T likes Biology and he is second to the left of O.

The person who likes History is not next to M or O.

The person who likes Physics sits fifth to the left of D, who likes Hindi.

 

Q. 82 Which of the following pairs sit between L and T when counted in anti-clockwise direction, starting from T?

A. C,O

B. K,D

C. V,C

D. M,O

E. Other than the given options

 

Q. 83 Who likes Physics?

A. O

B. T

C. V

D. D

E. Other than the given options

 

Q. 84 Four of the following five are alike a certain way and so form a group. Which is the one that does not belong to that group?

A. M

B. T

C. L

D. D

E. C

 

Q. 85 Who sits second to the left of the one who likes ‘History’?

A. V

B. M

C. One who likes Physics

D. Both 1) and 3)

E. Other than the given options

 

Q. 86 Which of the following does not match correctly?

A. V-Facing towards the centre – Physics

B. T- Facing outward the centre – Biology

C. C- Facing towards the centre – Civics

D. O- Facing outwards the cenntre – History

E. All except 2)

 

Questions: 87 – 90

Study the following information carefully and answer the questions given below:

Eight friends Seema, Suresh, Mahesh, Sudhir, Karan, Geeta, Raj and Sushil are sitting around a circular table. Three of them are facing outward the centre and five are facing towards the centre. Each one of them likes a different colour viz, Blue, green, Violet, White, Yellow, Pink, Red and Black. Suresh and Raj are facing towards the centre and the person who likes Green is second to the left of Raj.

Seema and Raj are facing each other and facing the same direction as Suresh.

Sudhir faces outward and sits opposite Suresh, who likes Black colour.

Geeta and Seema sit next to each other.

Geeta who faces outward sits opposite to one who likes Blue colour.

Yellow is liked by one of the two girls.

Mahesh sits third to the right of Geeta and likes Violet colour.

Geeta likes Pink colour

Sushil sits opposite Mahesh and Sudhir, second to the right of the one who likes Blue colour.

Raj likes Red colour and Sudhir likes whited.

Suresh sits between Seema and Mahesh.

 

Q. 87 Who sits fifth to the right of Geeta?

A. Mahesh

B. Karan

C. Raj

D. Suresh

E. Other than the given options

 

Q. 88 Three are alike in a certain way while one is not, find the odd one.

A. Raj-Seema

B. Suresh-Sudhir

C. Karan-Geeta

D. Seema-Mahesh

E. Other than the given options

 

Q. 89 Who among the following likes ‘Blue’ colour?

A. Geeta

B. Raj

C. Mahesh

D. Sushil

E. Karan

 

Q. 90 How many person sit between the person who likes White colour and the person who likes Black colour?

A. Two

B. Three

C. Four

D. None

E. Other then the given options

 

Questions: 91 – 95

Study the following information carefully and answer the questions given below:

There are eight persons viz. D, E, R, N, P, T, V and A sitting around a square table. They have a different professions viz. Engineer, Soldier, Teacher, Pilot, Artist, Doctor, Politician and Player but not necessarily in the same order. Four of them sit on the middle of the four sides while four of them sit on the four corners of the square table. All persons who sit at the four corners are facing the centre except one, while those who sit in the middle of the sides are facing outward the centre except one. T is neither Politician nor a Pilot. E is immediate left to the Player. N is not facing towards the centre. Soldier and Engineer are the neighbours of Doctor. D is a Teacher but not facing towards the centre. The Politician is the neighbour of both E and Soldier. Doctor is not facing towards the centre. R and A are facing each other but none of them is at middle of the sides. The Player is facing towards the centre but not sitting in the middle of the any side of the table. E sits second to the right of P, who is Soldier. Z is not the neighbour of either E or P and sits second to the right of N.V is between Artist and teacher.

 

Q. 91 Who among the following is a Pilot?

A. T

B. N

C. R

D. D

E. Z

 

Q. 92 Who among the following sits exactly between the Player and Politician?

A. E

B. Teacher

C. P

D. Both 1) and 2)

E. Artist

 

Q. 93 The profession of N is

A. Artist

B. Pilot

C. Doctor

D. Teacher

E. Player

 

Q. 94 How many person (s) sit (s) between V and the Doctor?

A. None

B. Three

C. Two

D. One

E. Other than the given options

 

Q. 95 Four of the following five are alike in a certain way, Find the odd man out?

A. ZT

B. PR

C. VE

D. ZD

E. RE

 

Questions: 96 – 100

Study the following information carefully and answer the questions given below:

In a certain code language “all players are energetic” is written as “zee dp zn nt” . “are they really energetic” is written as “mb nt re zn”, “related to all players” is written as “st zee dp ta” , discipline is related to game”, is written as “gt ta no st np” , ‘game is really to enjoy is written as ‘np re gt ta fr’.

 

Q. 96 Which of the following is the code for ‘enjoy’?

A. ta

B. st

C. gt

D. fr

E. no

 

Q. 97 Which of the following does ‘dp zn nt’ stand for?

A. all are energetic

B. players are energetic

C. they are players

D. all are players

E. Cannot be determined

 

Q. 98 Which of the following is the code for ‘discipline’?

A. no

B. st

C. gt

D. re

E. mb

 

Q. 99 If ‘the discipline is enjoyed’ is written as ‘kp no np fr’ then what is the code for ‘ the game to enjoy’?

A. no fr ta kp

B. kp gt ta fr

C. ta fr no gt

D. mb st gt nt

E. kp gt tec fr

 

Q. 100 Which of the following stand for ‘zee’?

A. all

B. player

C. energetic

D. players

E. Either 1 or 4

 

 

Answer Sheet
Question 1 2 3 4 5 6 7 8 9 10
Answer D A C E B A E B A B
Question 11 12 13 14 15 16 17 18 19 20
Answer A A D C C B D A B B
Question 21 22 23 24 25 26 27 28 29 30
Answer D A E B B D C A A E
Question 31 32 33 34 35 36 37 38 39 40
Answer B C E D B D B A E A
Question 41 42 43 44 45 46 47 48 49 50
Answer C B B A B C D A D C
Question 51 52 53 54 55 56 57 58 59 60
Answer D C A C B C D A D E
Question 61 62 63 64 65 66 67 68 69 70
Answer E A B A E B E D A C
Question 71 72 73 74 75 76 77 78 79 80
Answer B D A D E B D B C A
Question 81 82 83 84 85 86 87 88 89 90
Answer C B C B D E C D E B
Question 91 92 93 94 95 96 97 98 99 100
Answer E D C B B D E A B E

IBPS Clerk 2016 Mains Previous Year Paper

IBPS Clerk 2016 Mains

Section

Logical Reasoning

Questions

40 Questions (1 – 40)

Marks

40

English

40 Questions (41 – 80)

40

Quantitative Aptitude

30 Questions (81 – 110)

30

Data Interpretation

10 Questions (111 – 120)

10

General Knowledge

40 Questions (121 – 160)

40

Computer Aptitude

40 Questions (161 – 200)

40

Q. 1 How many such vowels are there in the given arrangement which are immediately preceded as well as immediately followed by a consonant?

A. One

B. Two

C. Three

D. Four

E. More than four

 

Q. 2 If all the vowels are deleted from the given arrangement, which of the following will be thirteenth from the right end?

A. K

B. G

C. M

D. D

E. None of these

 

Q. 3 How many letters are there between the letter that is twentieth from the left end and the letter that is sixteenth from the right end, in the English alphabetical series?

A. Five

B. Four

C. Six

D. Three

E. None of these

 

Q. 4 Which of the following is fifth to the right of the seventeenth from the left end of the given arrangement?

A. L

B. U

C. H

D. D

E. None of these

 

Q. 5 If all the vowels are changed to the next letter of the English alphabetical series, how many Bs will be there in the above arrangement? (Including the Bs already given in the arrangement)

A. Three

B. Two

C. Four

D. Six

E. Five

 

Q. 6 Four of the following five are alike in a certain way and hence form a group. Which of the following does not belong to that group?

A. 49

B. 36

C. 16

D. 27

E. 25

 

Q. 7 The positions of the first and fourth letters of the word ABSENT are interchanged, similarly the positions of the second and fifth letters and third and sixth letters are interchanged. In the new arrangement thus formed, how many letters are there between the letter which is third from the right and the letter which is first from the left, in the English alphabetical order?

A. Two

B. Four

C. Five

D. Three

E. None of these

 

Q. 8 Which of the following will come in the place of the question mark (?) in the following series based on the English alphabetical order?

BZ WU RP ? HF CA

A. LJ

B. MK

C. NL

D. KI

E. KJ

 

Q. 9 ‘Bricks’ are related to ‘Walls’ in the same way as ‘Trees’ are related to

A. Roots

B. Branches

C. Forests

D. Plants

E. Green

 

Q. 10 In a class of 40 children, Saurabh’s rank is eighth from the top. Mamta is five ranks below Saurabh. What is Mamta’s rank from the bottom?

A. 27th

B. 29th

C. 28th

D. 26th

E. Cannot be determined

 

Questions: 11 – 15

Study the given information carefully and answer the given questions.

J, K, L, M, N and O are sitting in a straight line facing North, not necessarily in the same order.

J is sitting fourth to the left of O and O is sitting at the extreme end of the line. M is sitting third to the right of N. K is not an immediate neighbour of J.

 

Q. 11 If all the persons are made to sit in an alphabetical order from left to right, the positions of how many will remain unchanged as compared to their original seating position?

A. One

B. Two

C. Three

D. More than three

E. None of these

 

Q. 12 What is the position of N with respect to J?

A. Second to the left

B. Third to the right

C. Second to the right

D. Immediate right

E. Immediate left

 

Q. 13 How many people are sitting between L and O?

A. One

B. Two

C. Three

D. More than three

E. None of these

 

Q. 14 Which of the pairs represent the people sitting at the extreme ends of the line?

A. JL

B. KJ

C. ML

D. MJ

E. None of these

 

Q. 15 Four of the following five are alike in a certain way based on their seating positions in the given arrangement and so form a group. Which one does not belong to the group?

A. LJ

B. OM

C. KN

D. OK

E. NJ

 

Q. 16 Starting from point X, Joy walked 15 m towards the West. He turned left and walked 20 m. He took another left and walked 15 m. After which he took a right turn and walked for another 12 m. How far is Joy from point X, if he faces North?

A. 27 m

B. 35 m

C. 32 m

D. 42 m

E. None of these

 

Q. 17 How many such pairs of letters are there in the word DECAYS, each of which has as many letters between them in the word (in both forward and backward directions) as they have between them in the English alphabetical series?

A. One

B. Two

C. Three

D. None

E. More than three

 

Q. 18 In the question given below, two statements are given followed by two conclusions numbered I and II. You have to take the two statements to be true even if they seem to be at variance from the commonly known facts and then decide which of the given conclusions logically follows from the given statements disregarding the commonly known facts.

Statements:

All animals are birds.

All birds are crows.

Conclusions:

I. All animals are crows.

II. All crows are birds.

A. Only conclusion I follows

B. Only conclusion II follows

C. Either conclusion I or II follows

D. Neither conclusion I nor II follows

E. Both conclusions I and II follow

 

Q. 19 In the question given below, two statements are given followed by two conclusions numbered I and II. You have to take the two statements to be true even if they seem to be at variance from the commonly known facts and then decide which of the given conclusions logically follows from the given statements disregarding the commonly known facts.

Statements:

Some tricks are magic.

All magic are true.

Conclusions:

I. There is a possibility that are tricks are true.

II. There is a possibility that all magic are tricks.

A. Only conclusion I follows

B. Only conclusion II follows

C. Either conclusion I or II follows

D. Neither conclusion I nor II follows

E. Both conclusions I and II follow

 

Q. 20 In the question given below, two statements are given followed by two conclusions numbered I and II. You have to take the two statements to be true even if they seem to be at variance from the commonly known facts and then decide which of the given conclusions logically follows from the given statements disregarding the commonly known facts.

Statements:

All e-mails are messages.

Some messages are letters.

Conclusions:

I. At least some letters are e-mails.

II. Some messages are not e-mails.

A. Only conclusion I follows

B. Only conclusion II follows

C. Either conclusion I or II follows

D. Neither conclusion I nor II follows

E. Both conclusions I and II follow

 

Q. 21 In the question given below, two statements are given followed by two conclusions numbered I and II. You have to take the two statements to be true even if they seem to be at variance from the commonly known facts and then decide which of the given conclusions logically follows from the given statements disregarding the commonly known facts.

Statements:

Some threads are cottons.

Some cottons are nylons.

Conclusions:

I. All nylons are threads.

II. At least some nylons are threads.

A. Only conclusion I follows

B. Only conclusion II follows

C. Either conclusion I or II follows

D. Neither conclusion I nor II follows

E. Both conclusions I and II follow

 

Q. 22 In the question given below, two statements are given followed by two conclusions numbered I and II. You have to take the two statements to be true even if they seem to be at variance from the commonly known facts and then decide which of the given conclusions logically follows from the given statements disregarding the commonly known facts.

Statements:

Some clocks are watches.

No clock is a wall.

Conclusions:

I. All walls are watches.

II. There is a possibility that all watches are clocks.

A. Only conclusion I follows

B. Only conclusion II follows

C. Either conclusion I or II follows

D. Neither conclusion I nor II follows

E. Both conclusions I and II follow

 

Questions: 23 – 27

These questions are based on the following three-digit numbers:

437 254 829 147 563

 

Q. 23 If all the digits of each of the numbers are arranged in an ascending order (within the number) which number will be the second highest?

A. 254

B. 437

C. 563

D. 147

E. 829

 

Q. 24 If all the three digits of each of the numbers are added, the total of which of the following will be the second lowest?

A. 437

B. 829

C. 147

D. 254

E. 563

 

Q. 25 If 1 is subtracted from the third digit of each of the numbers, how many numbers will be completely divisible by 3?

A. One

B. Two

C. Three

D. More than three

E. None

 

Q. 26 Which of the following is the product of the third digit of the lowest number and the second digit of the second highest number?

A. 42

B. 21

C. 35

D. 24

E. 6

 

Q. 27 If 1 is added to the second digit of each of the numbers, how many numbers will have more than one even digit (same or different even digit)?

A. One

B. Two

C. Three

D. More than three

E. None

 

Questions: 28 – 32

Study the following information carefully and answer the given questions:

A, B, C, D, E, F, G and H are sitting around a circular table, facing the centre. G is sitting third to the right of F. H is not an immediate neighbour of F and G. A is sitting second to the right of H. B is sitting third to the right of C and C is not an immediate neighbour of G. D is not an immediate neighbour of H or C.

 

Q. 28 Who is sitting exactly between H and A?

A. B

B. G

C. C

D. E

E. None of these

 

Q. 29 Four of the following five are alike in a certain way based on their seating positions in the given arrangement and so form a group. Which one does not belong to the group?

A. FD

B. HG

C. FC

D. BG

E. EA

 

Q. 30 What will come in place of the question mark (?) based upon the given seating

Arrangement?

AF DG CA ?

A. HC

B. DG

C. GE

D. BD

E. EH

 

Q. 31 How many persons are sitting between B and E starting from B in an anti-clockwise direction?

A. One

B. Two

C. Three

D. Four

E. None

 

Q. 32 What is the position of E with respect to A?

A. Third to the left

B. Third to the right

C. Second to the left

D. Second to the right

E. Immediate left

 

Q. 33 If it is possible to make only one meaningful word from the fourth, sixth, ninth and eleventh letters of the word CONTAMINATE, then the second letter from the left is your answer. If o such word can be formed then your answer is X and if more than one such word can be formed your answer is Y.

A. X

B. T

C. M

D. A

E. Y

 

Q. 34 In a certain code language, BRIDGE is written as DUKGIH and NUMBER is written as PXOEGU. How will CUSTOM be written in the same code language?

A. EWUVQO

B. EXUWQP

C. FXVWRP

D. EZUYQR

E. None of these

 

Q. 35 In the question, relationship between different elements is shown in the statements. The statements are followed by two conclusions.

Statements:

F>=K>G>+H=IConclusions

I. F>=H

II. G>J

A. Only conclusion I is true

B. Only conclusion II is true

C. Either conclusion I or II is true

D. Neither conclusion I nor II is true

E. Both conclusions I and II are true

 

Q. 36 In the question, relationship between different elements is shown in the statements. The statements are followed by two conclusions.

Statements:

F>=K>G>+H=IConclusions

I. G>=J

II. I<=K

A. Only conclusion I is true

B. Only conclusion II is true

C. Either conclusion I or II is true

D. Neither conclusion I nor II is true

E. Both conclusions I and II are true

 

Q. 37 In the question, relationship between different elements is shown in the statements. The statements are followed by two conclusions.

Statements:

J>K>=M>=R=T

Conclusions

I. T>K

II. M

A. Only conclusion I is true

B. Only conclusion II is true

C. Either conclusion I or II is true

D. Neither conclusion I nor II is true

E. Both conclusions I and II are true

 

Questions: 38 – 40

Study the following information carefully and answer the given questions.

In a certain code language,

‘want to start business’ is written as ‘tu ja na re’.

‘ business difficult to begin’ is written as ‘la re ke tu’.

‘business difficult and demanding’ is written as ‘ba tu da la’

‘demanding business risky start’ is written as ‘tu ja da sa’

 

Q. 38 What is the code for ‘begin’?

A. re

B. ke

C. la

D. tu

E. Cannot be determined

 

Q. 39 Which of the following represents ‘difficult start’?

A. na la

B. la tu

C. ke ja

D. ja la

E. da ja

 

Q. 40 What is the code for ‘want’?

A. na

B. tu

C. ja

D. re

E. Either ‘na’ or ‘ja’

 

Questions: 41 – 50

Read the following passage carefully and answer the questions given below it in the context of the passage.

If the Central Advisory Board on Education was hasty in congratulating itself on its clarion call to the government to raise the expenditure on education to 6% of the national income, it ought to remember that this demand was first raised by the Kothari Commission an entire quarter of a century ago. At present, only a little more than 3% of GNP is spent on this vital need and it is unrealistic to expect that it can be doubled overnight. For that matter the decline in allocations for education is mirrored in the Five Year Plans. While the First devoted on all-time high of 7.86%, it had progressively declined to 2.59% in the Sixth Plan and rose marginally to 3.55% in the next. As the advisory board recognises a paucity of funds will render it impossible to meet long overdue targets like the universalisation of primary education and the eradication of illiteracy. At the same time, when the allocation in the Eighth Plan is decided later this year, there will be several conflicting claims on scarce resources and in all likelihood education, like health and family welfare, will get short shrift. And yet, there is ‘no room for despair’. Founds are an essential requisite, but by no means the only one. Even within existing constraints, there is a lot that both the Centre and State governments can do. For example, each State can reduce the amount it spends out of its total education budget on staff, which has reached as much as 92% in some instances. Besides, there is a strong case for revising fees in some professional education institutions. Both medical colleges and IITs could charge a great deal more than they now do and most entrants would be happy to bear the increased burden, considering what they would otherwise spend on similar education abroad or in colleges charging capitation fees. IITs now charge a measly Rs. 500 a year from students, presumably in the belief that these institutions are engaged in an altogether nobler mission that in the pursuit of greater financial independence, although surveys show almost all such students hail from reasonably well-off families. Another reform, which many institutions could consider an many already do, is to hold morning and evening classes, which amounts to shifts. Given the pressure on all resources, physical as well as financial, there is no reason why all such innovations should not be experimented with – provided, of course, poor students do not suffer as a consequence and standards as a whole are maintained.

 

Q. 41 Which of the following is the recommendation of the Central Advisory Board regarding allocations on education?

A. 6% increase from that of the Seventh Plan

B. 3% of GNP

C. Reduction of amount spent on staff

D. Increase in fee in professional institutions

E. None of the above

 

Q. 42 Which of the following conveys the meaning of ‘no room for despair’?

A. No scope for improvement

B. Not much need to worry

C. No place for hope

D. No need to panic

E. No chance for increase

 

Q. 43 Which of the following is false in the context of the passage?

A. Some innovations in educations should be tried out.

B. A high percentage out of the total budget of education is spent on staff.

C. IITs, should charge capitation fees.

D. Professional education in foreign countries is expensive.

E. None of the above

 

Q. 44 According to the passage, which sector, apart from education, gets small budgetary allocation?

A. Adult literacy

B. Child welfare

C. Health and family welfare

D. Not mentioned in the passage

E. None of the above

 

Q. 45 The need for 6% of the national income to be spent on education was earlier raised around which of the following years?

A. 1950

B. 1955

C. 1960

D. 1965

E. 1970

 

Q. 46 Which of the following is one of the long overdue target of education?

A. Free primary education

B. Higher education

C. 100% literacy

D. Spread of education in rural areas

E. Higher allocation for education

 

Q. 47 Which of the following is true in the context of the passage?

A. The demand for 6% allocation for education is unrealistic.

B. Funds are not essential for education.

C. Medical colleges charge less fee than IITs.

D. Most of the students in IITs are from poor families

E. There are few two shifts institutions in our country.

 

Q. 48 In which of the Five Year Plans was the allocation for education the highest?

A. First

B. Second

C. Sixth

D. Seventh

E. None of these

 

Q. 49 According to the passage, in the Eighth Plan, which of the following is the likelihood regarding allocation on education?

A. It would be drastically reduced.

B. It would be substantially increased.

C. 6% of the national income will be allocated.

D. There may not be any increase.

E. The expenditure on staff would be reduced.

 

Q. 50 Within the budgetary constraint, which of the following has been recommended to be done?

A. Spend less on professional education

B. Discontinue some programmes

C. Admit less number of students

D. Charge capitation fee

E. Increase fee for professional education

 

Questions: 51 – 55

Rearrange the following five sentences A, B, C, D and E in the proper sequence so as to form a meaningful paragraph, then answer the questions given below them.

A. Many consider it wrong to blight youngsters by recruiting them into armed forces at a young age.

B. It is very difficult to have an agreement on an issue when emotions run high.

C. The debate has come up whether is right or wrong.

D. In many countries military service is compulsory for all.

E. Some of these detractors of compulsory draft are even very angry.

 

Q. 51 Which sentence should come FOURTH in the paragraph?

A. A

B. B

C. C

D. D

E. E

 

Q. 52 Which sentence should come FIRST in the paragraph?

A. A

B. B

C. C

D. D

E. E

 

Q. 53 Which sentence should come LAST in the paragraph?

A. A

B. B

C. C

D. D

E. E

 

Q. 54 Which sentence should come THIRD in the paragraph?

A. A

B. B

C. C

D. D

E. E

 

Q. 55 Which sentence should come SECOND in the paragraph?

A. A

B. B

C. C

D. D

E. E

 

Q. 56 Given below are sentences in which two spaces have been left blank. You are to choose from among the given choices, the pair of words which would fit the two blanks in the given order.

He was initially __________ at the suggestion but was soon ___________ it himself.

A. frowning; rejecting

B. thrilled; propogating

C. shocked; advocating

D. impressed; negating

E. suspicious; trusting

 

Q. 57 Given below are sentences in which two spaces have been left blank. You are to choose from among the given choices, the pair of words which would fit the two blanks in the given order.

The Minister felt that the _______ made by the Committee was ________ even though similar schemes had worked earlier.

A. election; acceptable

B. choice; profitable

C. decision; gainful

D. recommendation; infeasible

E. acceptance; approved

 

Q. 58 Given below are sentences in which two spaces have been left blank. You are to choose from among the given choices, the pair of words which would fit the two blanks in the given order.

Nabeesa was not _______ by the criticism and paid no _______ even when her best friend talked against her.

A. threatened; warning

B. troubled; mind

C. deterred; heed

D. bothered; attention

E. shaken; indication

 

Q. 59 Given below are sentences in which two spaces have been left blank. You are to choose from among the given choices, the pair of words which would fit the two blanks in the given order.

The activities of the association have ________ from the _________ objectives set for it in the initial years.

A. emerged; total

B. deviated; original

C. grown; simple

D. details; grand

E. increased; perverse

 

Q. 60 Given below are sentences in which two spaces have been left blank. You are to choose from among the given choices, the pair of words which would fit the two blanks in the given order.

The _______ imposed for non-payment was too ________ for it to bring in improvement in collection.

A. penalty; low

B. fine; severe

C. punishment; harsh

D. toll; simple

E. damage; cruel

 

Q. 61 Read the sentence to find out whether there is any grammatical error in it. The error, if any, will be in one part of the sentence. The number of that part is the answer. If there is no error, the answer is (e). (Ignore errors of punctuation, if any)

No hill station (a)/is as beautiful (b)/as Darjeeling (c)/with its scenic beauty. (d)

A. (a)

B. (b)

C. (c)

D. (d)

E. (e)

 

Q. 62 Read the sentence to find out whether there is any grammatical error in it. The error, if any, will be in one part of the sentence. The number of that part is the answer. If there is no error, the answer is (e). (Ignore errors of punctuation, if any)

The chief idea of (a)/a very common type of traveller (b)/is to see as many objects (c)/of interest as he possibly could. (d)

A. (a)

B. (b)

C. (c)

D. (d)

E. (e)

 

Q. 63 Read the sentence to find out whether there is any grammatical error in it. The error, if any, will be in one part of the sentence. The number of that part is the answer. If there is no error, the answer is (e). (Ignore errors of punctuation, if any)

These Acts were (a)/pushed through the Parliament (b)/in spite of opposition (c)/but with little modification. (d)

A. (a)

B. (b)

C. (c)

D. (d)

E. (e)

 

Q. 64 Read the sentence to find out whether there is any grammatical error in it. The error, if any, will be in one part of the sentence. The number of that part is the answer. If there is no error, the answer is (e). (Ignore errors of punctuation, if any)

One of the most (a)/widely spread (b)/bad habit (c)/is the use of tobacco. (d)

A. (a)

B. (b)

C. (c)

D. (d)

E. (e)

 

Q. 65 Pick out the word that is most nearly the same in meaning to the given words.

Mirrored

A. Image

B. Imitated

C. Reflected

D. Copied

E. Followed

 

Q. 66 Pick out the word that is most nearly the same in meaning to the given words.

Hail from

A. Greet with

B. Born in

C. Admitted from

D. Come from

E. Are known as

 

Q. 67 Pick out the word that is most nearly the same in meaning to the given words.

Progressively

A. Promoting

B. Liberally

C. Intermittently

D. Continuously

E. Constantly

 

Q. 68 Pick out the word that is most nearly the opposite in meaning to the given words.

Unrealistic

A. Reasonable

B. Actual

C. Visionary

D. Intelligent

E. Natural

 

Q. 69 Pick out the word that is most nearly the opposite in meaning to the given words.

Vital

A. Dead

B. Unimportant

C. Lead

D. Outer

E. Peripheral

 

Q. 70 Pick out the word that is most nearly the opposite in meaning to the given words.

Paucity

A. Want

B. Richness

C. Presence

D. Scarcity

E. Surplus

 

Questions: 71 – 80

In the following passage there are blanks, each of which has been numbered. These numbers are printed below the passage and against each, five words are suggested, one of which fits the blank appropriately. Find out the appropriate words. Conventional medical wisdom does not advise taking (71) vitains as pills and capsules. On the contrary, there was a strong lobby which (72) the supplementary intake of vitamins. This used to leave the (73) person confused. However, modern research is (74) a surprise. More and more scientists are beginning to (75) that (76) medical views on vitamins have been (77). Evidence shows that (78) extra vitamins play a role in assuring vitality and (79) health than was (80) thought.

 

Q. 71 Find the appropriate word to fit the corresponding blank in the passage.

A. some

B. extra

C. insoluble

D. strong

E. all

 

Q. 72 Find the appropriate word to fit the corresponding blank in the passage.

A. facilitated

B. criticised

C. communicated

D. projected

E. advocated

 

Q. 73 Find the appropriate word to fit the corresponding blank in the passage.

A. medical

B. sick

C. scientific

D. average

E. educated

 

Q. 74 Find the appropriate word to fit the corresponding blank in the passage.

A. trying

B. manufacturing

C. practicing

D. holding

E. inventing

 

Q. 75 Find the appropriate word to fit the corresponding blank in the passage.

A. trust

B. consider

C. refute

D. forget

E. suspect

 

Q. 76 Find the appropriate word to fit the corresponding blank in the passage.

A. indigent

B. all

C. traditional

D. modern

E. allopathic

 

Q. 77 Find the appropriate word to fit the corresponding blank in the passage.

A. practical

B. limited

C. feasible

D. correct

E. confusing

 

Q. 78 Find the appropriate word to fit the corresponding blank in the passage.

A. marketing

B. manufacturing

C. swallowing

D. taking

E. buying

 

Q. 79 Find the appropriate word to fit the corresponding blank in the passage.

A. pleasurable

B. manly

C. optimal

D. stronger

E. useful

 

Q. 80 Find the appropriate word to fit the corresponding blank in the passage.

A. previously

B. occasionally

C. seriously

D. even

E. now

 

Q. 81 What should come at the place of question mark (?) in the following questions?

7059 – 2350 + 1936 = ? x 50

A. 123.6

B. 132.3

C. 132.6

D. 123.9

E. 132.9

 

Q. 82 What should come at the place of question mark (?) in the following questions?

A. (a)

B. (b)

C. (c)

D. (d)

E. (e)

 

Q. 83 What should come at the place of question mark (?) in the following question?

A. (a)

B. (b)

C. (c)

D. (d)

E. (e)

 

Q. 84 What should come at the place of question mark (?) in the following question?

683.46 – 227.39 – 341.85 = ?

A. 114.22

B. 141.22

C. 144.22

D. 112.22

E. None of these

 

Q. 85 What is 786 times of 964?

A. 759276

B. 7498

C. 75416

D. 75770

E. None of these

 

Q. 86 What approximate value should come in place of the question mark (?) in the following question? (You are not expected to calculate the exact value).

16.003 x 27.998 – 209.010 = ?

A. 150

B. 200

C. 75

D. 240

E. 110

 

Q. 87 What approximate value should come in place of the question mark (?) in the following question? (You are not expected to calculate the exact value).

125.009 + 69.999 + 104.989 = ?

A. 420

B. 300

C. 285

D. 415

E. 325

 

Q. 88 What approximate value should come in place of the question mark (?) in the following question? (You are not expected to calculate the exact value).

840.003/23.999 = ?

A. 47

B. 8

C. 35

D. 18

E. 23

 

Q. 89 What will come in place of question mark (?) in the given number series?

2 4 10 ? 82 244

A. 30

B. 48

C. 28

D. 46

E. 34

 

Q. 90 What will come in place of question mark (?) in the given number series?

7 8 4 13 -3 22 ?

A. -7

B. -10

C. -12

D. -14

E. -9

 

Q. 91 The difference between a two-digit number and the number obtained by interchanging the two digits of the number is 18. The sum of the two digits of the number is 12. What is the product of the digits of two-digit number?

A. 35

B. 27

C. 32

D. Can’t be determined

E. None of these

 

Q. 92 The average height of the basketball team A is 5 ft 11 inches and that of B is 6 ft 2 inches. There are 20 players in team A and 18 players in team B. The overall average height is

A. 72.42 inches

B. 72 inches

C. 70.22 inches

D. 70 inches

E. None of these

 

Q. 93 A water pipe is cut into two pieces. The longer piece is 70% of the length of the pipe. By how much percentage is the longer piece longer than the shorter piece?

A. 140%

B. 400/3%

C. 40%

D. 300/4%

E. None of these

 

Q. 94 In an examination, the marks obtained by Shantanu is 40% less than the marks obtained by Kamal, then marks obtained by Kamal is how much percent more than the marks obtained by Shantanu?

A. 55 2/3 %

B. 44 3/5 %

C. 33 1/3 %

D. 66 2/3 %

E. None of these

 

Q. 95 Neeta got profit of 10% on selling an article in Rs. 220. To get the profit of 30%, she should sell the article in how many rupees?

A. 220

B. 230

C. 260

D. 280

E. None of these

 

Q. 96 While selling, a businessman allows 40% discount on the marked price and there is a loss of 30%. If it is sold at the marked price, profit percent will be

A. 10%

B. 20%

C. 16 2/3%

D. 16 1/3%

E. None of these

 

Q. 97 In a certain time, a sum becomes 4 times at the rate of 5% per annum. At what rate of simple interest, the same sum becomes 8 times in the same duration?

A. 12 2/3%

B. 11 3/5%

C. 11 2/3%

D. 12 3/5%

E. None of these

 

Q. 98 A woman bought eggs at Rs. 30 per dozen. The selling price per hundred so as to gain 12% will be

A. Rs. 280

B. Rs. 250

C. Rs. 300

D. Rs. 360

E. None of these

 

Q. 99 A certain sum at compound interest amounts to Rs. 12960 in 2 years and Rs. 13176 in 3 years. Find the rate percent per annum.

A. 1 1/3%

B. 2 1/3%

C. 1 2/3%

D. 2 2/3%

E. None of these

 

Q. 100 Vandana bought a watch for Rs. 600 and sold it the same day for Rs. 688.50 at a credit of 9 months and this way she gained 2%. Find the rate of interest per annum.

A. 16 2/3%

B. 15 2/3%

C. 11 2/3%

D. 5 2/3%

E. None of these

 

Q. 101 35% of a number is two times 75% of another number. What is the ratio between the first and the second number, respectively?

A. 35:6

B. 31:7

C. 23:7

D. 32:9

E. 30:7

 

Q. 102 Amit and Sudesh have invested in the ratio of 4:7. If both invested a total amount of Rs. 49500, then find the investment of Sudesh.

A. Rs. 31500

B. Rs. 1800

C. Rs. 31000

D. Rs. 18500

E. None of these

 

Q. 103 300 g of salt solution has 40% salt in it. How much salt should be added to make it 50% in the solution?

A. 40 g

B. 60 g

C. 70 g

D. 80 g

E. None of these

 

Q. 104 A tree is 12 m tall and casts an 8 m long shadow. At the same time, a flag pole casts a 100 m long shadow. How long is the flag pole?

A. 150 m

B. 200 m

C. 125 m

D. 115 m

E. None of these

 

Q. 105 If 5 boys take 7 hours to pack 35 toys, how many boys can pack 66 toys in 3 hours?

A. 26

B. 39

C. 45

D. 65

E. None of these

 

Q. 106 Two pipes A and B can fill a tank in 18 and 6 hours, respectively. If both the pipes are opened simultaneously, how much time will be taken to fill the tank?

A. 4.5 h

B. 7 h

C. 6 h

D. 10 h

E. 9 h

 

Q. 107 A bus covers a certain distance in 16 hours. It covers half the distance at 40 km/h and the rest at 60 km/h. Find the length of the journey.

A. 520 km

B. 448 km

C. 384 km

D. 768 km

E. None of these

 

Q. 108 If two trains start at the same time from points x and y towards each other and after crossing, they take 9 hours and 4 hours in reaching points y and x, respectively. Find the ratio of speeds of the first train to that of the second train.

A. 2:3

B. 3:2

C. 2:5

D. 5:3

E. None of these

 

Q. 109 A car runs at the speed of 50 km/h when not serviced and runs at 60 km/h, when serviced. After servicing, the car covers a certain distance in 6 h. How much time will the car take to cover the same distance when not serviced?

A. 8.2 h

B. 6.5 h

C. 8 h

D. 7.2 h

E. None of these

 

Q. 110 At present, Kavita is twice Sarita’s age. 8 years hence, the respective ratio between Kavita’s and Sarita’s ages will be 22:13. What is Kavita’s present age?

A. 26 yr

B. 18 yr

C. 42 yr

D. 36 yr

E. None of these

 

Questions: 111 – 115

The circle graph given here shows the spendings of a country on various sports during a particular year. Study the graph carefully and answer the questions given below it.

 

Q. 111 What percent of the total spendings is spent on Tennis?

A. 12.5%

B. 22.5%

C. 25%

D. 45%

E. None of these

 

Q. 112 How much percent more is spent on Hockey than that on Golf?

A. 27%

B. 35%

C. 37.5%

D. 75%

E. None of these

 

Q. 113 How much percent less is spent on Football than that on Cricket?

A. 22 2/9%

B. 27%

C. 33 1/3%

D. 75%

E. None of these

 

Q. 114 If the total amount spent on sports during the year was Rs. 2 crore, the amount spent on Cricket and Hockey together was

A. Rs. 800000

B. Rs. 8000000

C. Rs. 12000000

D. Rs. 16000000

E. None of these

 

Q. 115 If the total amount spent on sports during the year be Rs. 18000000, the amount spent on Basketball exceeds that on Tennis by

A. Rs. 250000

B. Rs. 360000

C. Rs. 375000

D. Rs. 410000

E. None of these

 

Questions: 116 – 120

Study the bar-graph and answer the questions.

 

Q. 116 In the year 2006, the people preferring to play Tennis is what percent of the people preferring to play Cricket, Football and Tennis together in that year?

A. 25 1/3%

B. 24 3/7%

C. 21 1/3%

D. 22 2/9%

E. 23 1/3%

 

Q. 117 From 2001 to 2006, what was the total number of people who preferred to play Football (in millions)?

A. 1525

B. 1620

C. 1730

D. 1800

E. 1925

 

Q. 118 The number of people preferring to play Tennis in 2006, is how many millions fewer than the number of people preferring to play Tennis in 2005?

A. 115

B. 120

C. 100

D. 97

E. 95

 

Q. 119 What is the respective ratio of the number of people preferring to play Cricket to the number of people, preferring to play Tennis in the year 2003?

A. 14:15

B. 15:13

C. 15:17

D. 13:15

E. 17:14

 

Q. 120 How many people (in millions) have preferred to play Cricket in all the years together?

A. 2050

B. 2000

C. 1850

D. 1750

E. 1600

 

Q. 121 The first Indian State to have Defence Industrial Park, is

A. Karnataka

B. Kerala

C. Maharashtra

D. Andhra Pradesh

E. Bihar

 

Q. 122 Professor Maurice Obstfeld will become Economic Counsellor and Director of the ______ research department in September.

A. IMF

B. City Bank

C. ADB

D. PNB

E. ICICI Bank

 

Q. 123 A 20-year old Indian-American ________ won the junior title at the latest United States Powerlifting Association National Championships in Las Vegas.

A. Jitesh Chhabra

B. Amitoj Chhabra

C. Amitoj Singh

D. Raj Singh

E. Jitesh Singh

 

Q. 124 Ambati Rayudu is related to

A. cricket

B. tennis

C. shooting

D. snooker

E. None of these

 

Q. 125 To safeguard small investors from high-risk products, the market regulator SEBI made a steep hike in the minimum investment size for any equity derivative product to Rs. 5 lakh from ______ currently.

A. Rs. 2 lakh

B. Rs. 1 lakh

C. Rs 3 lakh

D. Rs. 4 lakh

E. Rs. 4.5 lakh

 

Q. 126 _______ won the South Asian Basketball (SABA) Championship title 2015.

A. Germany

B. The USA

C. India

D. France

E. South Korea

 

Q. 127 Kotak Mahindra Bank has got a go-ahead from the Foreign Investment Promotion Board (FIPB) for its proposal to increase Foreign Direct Investment (FDI) in the bank to

A. 25%

B. 35%

C. 55%

D. 75%

E. 95%

 

Q. 128 Indian pair of Jwala Gutta and Ashwini Ponappa won the women’s doubles of

A. Canada Open Badminton Tournament 2015

B. Norway Chess Tournament 2015

C. World Railways Championship 2015

D. Tennis Championships 2015

E. Aegon Tennis Championships

 

Q. 129 President Pranab Mukherjee has nominated two Anglo-Indian Community members to the Lower House as per the provisions of Article-331 of Indian Constitution namely

I. Professor Richard Hay

II. Professor Richard Rostra

III. George Baker

IV. G. Richard

Which of the above are correct?

A. I and III

B. I and II

C. II and III

D. III and IV

E. I and IV

 

Q. 130 eMudhra offers variety of other services including

I. tax filing services

II. digital signing solutions

III. PAN Card online applications

IV. digital certificate authentication system

V. trusted cheap stamping

Select the correct answer using the codes given below:

A. I, II, III and IV

B. II, III, IV and V

C. I, III, IV and V

D. I, IV and V

E. All of these

 

Q. 131 Choose the correct statement(s):

A. YK Sabharwal was former Chief Justice of India

B. Justice Sabharwal was Chief Justice of India from November 1, 2005 to January 13, 2007

C. Justice Sabharwal was honoured with Padma Bhushan in 2006

D. Both (a) and (b)

E. All of the above

 

Q. 132 Arrange the following world’s biggest employers in chronological order (According to World Hunger List):

I. Walmart

II. McDonald’s

III. Indian Railways

IV. Indian Army

A. I, II, IV and III

B. III, II, IV and I

C. I, II, III and IV

D. II, III, I and IV

E. II, I, IV and III

 

Q. 133 First-time MLA ______, was sworn-in as a Cabinet Minister in the Delhi government on June, 2015 and given charge of the Law Department.

A. Jitendra Singh Tomar

B. Kapil Mishra

C. Arvind Kejriwal

D. Sanjay Singh

E. Intzar Pervez

 

Q. 134 China has permitted events to celebrate the first UN Yoga Day on

A. June 21

B. June 18

C. June 15

D. June 9

E. June 3

 

Q. 135 Satish Dhawan Space Centre or Sriharikota Range (SHAR) is a rocket launch centre operated by

A. IRSO

B. VSSC

C. SHAR

D. ISRO

E. IORS

 

Q. 136 Asian Development Bank (ADB) has appointed ______ and Bambang Susantono as Vice- President in two different roles.

A. Vijai Sharma

B. KV Chowdary

C. Diwakar Gupta

D. Sanjay Singh

E. Vijay Singh

 

Q. 137 Serena Williams (USA), won French Open 2015 women’s singles title. This was her _______ French Open title and overall 20th Grand Slam title.

A. first

B. second

C. third

D. fourth

E. fifth

 

Q. 138 Consider the following statements:

I. Axis Bank will issue co-branded cards to the users of Kochi Metro for paying the fares as well as using them for all kinds of merchandise including e-commerce.

II. An Agreement in this regard was signed in June, 2015 between KMRL and Axis Bank.

III. CMD of KMRL, Elias George and MD and CEO of Axis Bank Shikha Sharma signed the agreement.

Choose the correct statement(s).

A. I and II

B. II and III

C. All of these

D. I and III

E. Only III

 

Q. 139 According to Forbes magazine’s highest-paid athletes in the world, arrange the following in chronological order :

I. Floyd Mayweather

II. Mahendra Singh Dhoni

III. Roger Federer

A. II, III, I

B. I, III, II

C. I, II, III

D. II, I, III

E. III, II, I

 

Q. 140 Arrange the following country in chronological order (According to the World Hunger List) :

I. Pakistan

II. India

III. China

IV. Nigeria

A. I, II, IV, III

B. III, II, IV, I

C. I, III, IV, II

D. II, III, I, IV

E. II, I, IV, III

 

Q. 141 National Thermal Power Corporation (NTPC) in May, 2015 signed a pact with which State government for setting-up a 4000 MW power plant in the State?

A. Bihar

B. Jharkhand

C. Uttar Pradesh

D. Uttarakhand

E. Gujarat

 

Q. 142 GMR Infrastructure Limited has signed a Memorandum of Understandings (MoU) with Guizhou International Investment Corporation (GIIC) for developing Kakinada Special Economic Zone (SEZ) in

A. Rajasthan

B. Karnataka

C. Tamil Nadu

D. Madhya Pradesh

E. Andhra Pradesh

 

Q. 143 FDI in Railway Infrastructure sector has been opened to _____ FDI under automatic route.

A. 49%

B. 50%

C. 60%

D. 75%

E. 100%

 

Q. 144 A record number of ______ Indian-origin candidates including Keith Vaz, Priti Patel and Infosysy co-founder Narayana Murthy;s son-in-law were elected on May 8, 2015, to the British Parliament.

A. seven

B. eight

C. nine

D. ten

E. fifteen

 

Q. 145 MoneyGram is the second largest provider of money _____ in the world.

A. saving account

B. transfers

C. calculation software

D. payment way

E. None of these

 

Q. 146 Future Group is an Indian private conglomerate, headquartered in

A. Mumbai

B. Delhi

C. Hyderabad

D. Bangalore

E. Gwalior

 

Q. 147 The Indian government raised the limit of Foreign Direct Investment (FDI) in the pension sector to _____ in line with the FDI cap raised recently in the insurance sector.

A. 25%

B. 35%

C. 49%

D. 59%

E. 64%

 

Q. 148 Choose the correct statements regarding the Soil Health Card Scheme (SHC).

A. SHC has in it all basic and crop wise recommendations of fertilizers and nutrients.

B. Punjab has become the first ever State in the country to issue Soil Health Cards.

C. Checking of soil health on quarterly basis.

D. Both (a) and (b)

E. All of the above

 

Q. 149 Pradhan Mantri Jeevan Jyoti Bima Yojna (PMJJBY) and Pradhan Mantri Suraksha Bima Yojna (PMSBY) will cover

A. PMJJBY – all the savings account holders of the age group 18-50 for death due to any cause.

B. PMSBY will cover all the savings account holders of the age group 18 to 70 for accidental disability or death

C. The premium for PMJJBY is Rs. 330 per year.

D. The premium for PBSBY is Rs. 12

E. All of the above

 

Q. 150 The World Snooker Championship is a leading professional snooker tournament in terms of

I. Prize money

II. Ranking points

III. Prestige

Select the correct answer using the codes given below.

A. Only I

B. Only III

C. I and II

D. All of these

E. None of these

 

Q. 151 Cincinnati Masters, started on September 18, 1899, is the oldest ______ tournament in the United States (US) played in its original city.

A. football

B. tennis

C. chess

D. snooker

E. golf

 

Q. 152 India’s Aditi Chauhan has become the country’s _______ woman to play English league football.

A. first

B. second

C. third

D. fourth

E. fifth

 

Q. 153 Country’s largest lender State Bank of India launched a mobile wallet app in August, 2015, ________, in collaboration with Accenture and Mastercard.

A. SBI Amast

B. SBI Buddy

C. SBI Card M

D. SBI Mcard

E. SBI Dubby

 

Q. 154 Indian sociobiologist Raghavendra Gadagkar was awarded ______ highest civilian honour, the Cross of Order of the Merit.

A. Singapore’s

B. Bhutan’s

C. China’s

D. Germany’s

E. Britain’s

 

Q. 155 SBI Life Insurance has launched a new product, ______ on August 7, 2015, a plan which offers multiple benefits of savings and insurance cover for husband and wife under a single policy.

A. Smart Family

B. Smart Humsafar

C. Smart Policy

D. Smart Safar

E. Smart Insurance

 

Q. 156 Pakistan is the ______ in the world after Turkey and Serbia to get the status of CERN’s associate membership.

A. First

B. Second

C. Third

D. Fourth

E. Fifth

 

Q. 157 Choose the correct statement(s) regarding portal feature of Vidya Lakshmi :

A. Facility to apply to multiple Banks for Educational Loans.

B. Facility for Banks to download Students’ Loan Applications.

C. Facility for Banks to upload loan processing status.

D. Facility for Students to e-mail grievances/queries relating to Educational.

E. All of the above

 

Q. 158 Choose the correct statement(s) regarding six-nation SAFF Under-16 Championship :

A. Bangladesh Under-16 Football team win 3rd edition title of this trophy by India by 4-2 goals in tie breaker at the Sylhet Stadium.

B. The first edition of the tournament took place in Nepal in August, 2011.

C. Besides India and Bangladesh, the tournament was participated by Nepal, Sri Lanka, Afghanistan and the Maldives.

D. All of the above

E. None of the above

 

Q. 159 Subsidy is a benefit given by the government to groups or individuals usually in the form of

A. a cash payment

B. tax reduction

C. Discount rate

D. Both (a) and (b)

E. Both (b) and (c)

 

Q. 160 The Board of Control for Cricket in India (BCCI) announced on August 7, 2015 the formation of an ad-hoc committee for cricket administration (CA) in the State(s) of

A. Bihar

B. Uttarakhand

C. Rajasthan

D. Both (a) and (b)

E. Both (b) and (c)

 

Q. 161 A telephone number, a birth date, and a customer name are all examples of

A. a record

B. data

C. a file

D. a database

E. a folder

 

Q. 162 Which key is used in combination with another key to perform a specific task?

A. Function

B. Control

C. Arrow

D. Space bar

E. Shift

 

Q. 163 In addition to keying data directly into a data directly into a database, data entry can be done from a(n)

A. input form

B. table

C. field

D. data dictionary

E. tuple

 

Q. 164 If a new device is attached to a computer, such as a printer or scanner, its _______ must be installed before the device can be used.

A. buffer

B. driver

C. pager

D. server

E. maker

 

Q. 165 The software that allows users to surf the internet is called a/an

A. Search engine

B. Internet Service Provider (ISP)

C. Multimedia application

D. Browser

E. Cookie

 

Q. 166 ______ provides a common address space and routes the packets of data across the entire internet.

A. IP

B. TCP

C. UDP

D. ALU

E. FTP

 

Q. 167 What menu is selected to change the font and style?

A. Tools

B. File

C. Format

D. Edit

E. Insert

 

Q. 168 Primary memory is used by the

A. user

B. input device

C. CPU

D. output device

E. programmer

 

Q. 169 A ________ is a computer connected to two networks.

A. link

B. server

C. gateway

D. bridge way

E. client

 

Q. 170 Which of the following is not an operating system?

A. DOS

B. Oracle

C. Linux

D. Windows

E. Unix

 

Q. 171 A small amount of memory included in the processor for high speed access ID called

A. Register

B. Cache

C. RAM

D. ROM

E. Virtual

 

Q. 172 The ______ enables you to simultaneously keep multiple webpages open in one browser window.

A. Tab bar

B. pop-up helper

C. Tab row

D. Address bar

E. Tab column

 

Q. 173 The ________ button on the quick access toolbar allows you to cancel your recent commands or action.

A. Search

B. Cut

C. Document

D. Undo

E. Redo

 

Q. 174 What type of virus use computer hosts to reproduce it?

A. Time bomb

B. Worm

C. Melissa virus

D. Macro virus

E. Trojan

 

Q. 175 _____ is a signalling method that handles a relatively wide range of frequencies.

A. Broadband

B. DLF

C. Dial-up

D. Both (a) and (c)

E. Wireless

 

Q. 176 A disk content that is recorded at the time of manufacture and that cannot be changed or erased by the user

A. Read only

B. Memory only

C. Run only

D. Writes only

E. Search only

 

Q. 177 The OCR is used for the preparation of

A. electricity bills

B. insurance premium

C. telephone bills

D. gas bills

E. All of these

 

Q. 178 Which of the following unit is not used to count the speed of a printer?

A. CPM

B. DPI

C. PPM

D. BIT

E. LPM

 

Q. 179 The binary equivalent of decimal number 98 is

A. 1110001

B. 1110100

C. 1100010

D. 1111001

E. 0110110

 

Q. 180 This part of operating system manages the essential peripherals, such as the keyboard, screen, disk drives, and parallel and serial ports.

A. basic input/output system

B. secondary input/output system

C. peripheral input/output system

D. marginal input/output system

E. primary input/output system

 

Q. 181 Which of the following is not a type of key?

A. Alphabetic keys

B. Numeric keys

C. Function keys

D. Toggle keys

E. None of these

 

Q. 182 If a previously saved file is edited

A. it cannot be saved again

B. the changes will automatically be saved in the file

C. the file will only have to be saved again if it is more than one page in length

D. its name must be changed

E. the file must be saved again to store the changes

 

Q. 183 Which of the following statements is false concerning file names?

A. Files may share the same name or the same extension but not both.

B. Every file in the same folder must have a unique name.

C. File extension is another name for file type

D. The file extension comes before the dot (.) followed by the file name.

E. None of the above

 

Q. 184 What is gutter margin?

A. Margin that is added to the left margin when printing.

B. Margin that is added to right margin when printing.

C. Margin that is added to the binding side of page when printing.

D. Margin that is added to the outside of the page when printing

E. None of the above

 

Q. 185 By default, your document will print in _______ mode.

A. landscape

B. portrait

C. page set-up

D. print view

E. page preview

 

Q. 186 _____ is a procedure that requires users to enter an identification code and a matching password.

A. Paging

B. Logging on

C. Time sharing

D. Multitasking

E. Logging off

 

Q. 187 Computer programs are written in a high-level programming language; however, the human-readable version of a program is called

A. Cache

B. Instruction set

C. Source code

D. Word size

E. Object code

 

Q. 188 Which of the following refers to the memory in your computer?

A. RAM

B. DSL

C. USB

D. LAN

E. CPU

 

Q. 189 A series of instructions that tells a computer what to do and how to do it is called a

A. Program

B. Command

C. User response

D. Processor

E. Server response

 

Q. 190 Name of the executable file of MS Word is

A. MSWord

B. MSWord.exe

C. WinWord

D. WinWord.exe

E. Word.exe

 

Q. 191 _____ are viruses that are triggered by the passage of time or on a certain date.

A. Boot-sector viruses

B. Macro viruses

C. Time bombs

D. Worms

E. Trojan Horse

 

Q. 192 What is a backup?

A. Restoring the information backup

B. An exact copy of a system’s information

C. The ability to get a system up and running in the event of a system crash or failure

D. All of the above

E. None of the above

 

Q. 193 In a rung topology, the computer in possession of the _____ can transmit data.

A. packet

B. data

C. access method

D. token

E. hob

 

Q. 194 _______ is the act of copying or downloading a program from a network and making multiple copies of it.

A. Network piracy

B. Plagiarism

C. Software piracy

D. Site-license piracy

E. Hardware piracy

 

Q. 195 A device operating at the physical layer is called a

A. Bridge

B. Router

C. Repeater

D. Gateway

E. Network

 

Q. 196 Which of the following terms is just the connection of networks that can be joined together?

A. Virtual Private Network

B. Internet

C. Intranet

D. Extranet

E. Public network

 

Q. 197 An affordable technology that uses existing telephone lines to provide high-speed connections is called

A. ISDN

B. Microwave

C. Cable modem

D. DSL

E. ALU

 

Q. 198 The capacity of a communication channel is measured in

A. bandwidth

B. bit capacity

C. baud rate

D. data flow

E. store data

 

Q. 199 A relatively new technology that allows wireless connectivity

A. Bluetooth

B. Black tooth

C. Blue band

D. Broadband

E. Wi-FI

 

Q. 200 For large computer network which topology is used?

A. Star

B. Ring

C. Bus

D. Mesh

E. Tree

 

 

Answer Sheet
Question 1 2 3 4 5 6 7 8 9 10
Answer E D B A E D D C C C
Question 11 12 13 14 15 16 17 18 19 20
Answer A C B D E C B A E D
Question 21 22 23 24 25 26 27 28 29 30
Answer D B B C A A B C E C
Question 31 32 33 34 35 36 37 38 39 40
Answer B A E B D D B B D A
Question 41 42 43 44 45 46 47 48 49 50
Answer E B C C D C A A D E
Question 51 52 53 54 55 56 57 58 59 60
Answer E D B A A C D C B A
Question 61 62 63 64 65 66 67 68 69 70
Answer A D D C C D D A E E
Question 71 72 73 74 75 76 77 78 79 80
Answer B E D D B C E D D A
Question 81 82 83 84 85 86 87 88 89 90
Answer E C D A D D B C C D
Question 91 92 93 94 95 96 97 98 99 100
Answer A A B D C C C A C A
Question 101 102 103 104 105 106 107 108 109 110
Answer E A B A E A D A D D
Question 111 112 113 114 115 116 117 118 119 120
Answer A D C B A E D C B A
Question 121 122 123 124 125 126 127 128 129 130
Answer B A B A A C C A A A
Question 131 132 133 134 135 136 137 138 139 140
Answer D C B A D C C C B D
Question 141 142 143 144 145 146 147 148 149 150
Answer B E E D B A C E E D
Question 151 152 153 154 155 156 157 158 159 160
Answer B A B D B C E D D D
Question 161 162 163 164 165 166 167 168 169 170
Answer A B A B D A C C D B
Question 171 172 173 174 175 176 177 178 179 180
Answer B C D B A A E D C A
Question 181 182 183 184 185 186 187 188 189 190
Answer E E D C B B C A A D
Question 191 192 193 194 195 196 197 198 199 200
Answer C D D C C B D A A D

 

IBPS Clerk 2015 Prelims Previous Year Paper

IBPS Clerk 2015 Prelims paper 3

Section

Questions

Marks

English

30 Questions (1 – 30)

30

Logical Reasoning

30 Questions

30

Quantitative Aptitude

15 Questions

15

Data Interpretation

5 Questions (96 – 100)

5

Questions: 1 – 5

Directions (1-5) :

In the following questions, some parts of the sentences have errors and some are correct. Find out which part of a sentence has an error. The number of that part is the answer. If a sentence is free from error, choose option 5 i.e. No error.

 

Q. 1 The sentence is –

A. The king humbly

B. thanked the

C. noble deer for

D. saving his life.

E. No error

 

Q. 2 The sentence is –

A. The fierce lion and

B. the sly fox became friend

C. and decided to

D. hunt together.

E. No error

 

Q. 3 The sentence is –

A. The clever disciple who

B. knew the special verse

C. wanted to show how he was

D. better than the others.

E. No error

 

Q. 4 The sentence is –

A. When the bison had gone down

B. to the water hole to drink

C. the jackal goes up to the lion

D. and started speaking to him.

E. No error

 

Q. 5 The sentence is –

A. The turtle would have

B. loved to accompany

C. his friends, but

D. he could not fly.

E. No error

 

Questions: 6 – 15

Directions (6-15) :

Read the following passage carefully and answer the questions given below it. Certain words/phrases have been given in quotes to help you locate them while answering some of the questions.

There once lived an old man. He had three sons. One day the old man called his three sons and said, ‘My end is near! I wish to give you certain things that I have kept for you.’ He then gave a rooster to his eldest son, an axe to the second and a cat to the youngest. ‘These things may seem “worthless” but everything depends on how sensibly you use them’, said the old man. In a few days, the old man died. His sons decided to make their fortune using the gifts, they received from their father. So, the eldest son went wandering about with his rooster. He crossed many towns and cities until he reached an island where people didn’t know how to divide their time. ‘Perhaps, this should be the “right” place to use my rooster’ he thought. So he called out to the people and said, ‘Look! Here is an amazing creature; it has a ruby-red crown upon its head like a knight. It crowd four times during the night and when it crows for the last time, the sun soon rises. But, if it crows in broad daylight, then be careful, there will “certainly” be a change in weather’. The people of the island were amazed to see such a creature. ‘Is this creature up for sale?’ they asked. ‘O yes! And the price is as many gold coins as a donkey can carry’, he replied. ‘Oh that’s just a small price for such a precious and useful creature who will help in keeping track of time’, they replied and willingly gave him the gold coins. When the eldest son came home with his wealth, the other two were astonished. Now the second son went wandering about with his axe. He crossed many farms where labourers were carrying their own axes. At last he “chanced upon” an island where people knew nothing about axes. So, he began displaying his axe and started narrating its uses. People were amazed to see such a tool and agreed to buy it for a horse laden with as much gold as it could carry. Now it was the third son’s turn to see what he could get of his cat. He went about wandering through many town until he reached a place where mice had created a menace and no cat had ever been seen. ‘We are fed up of the mice. They are so many that they dance upon our tables and benches and gnaw whatever they catch hold of’, they complained. The youngest son let his cat free for sometime. She began her chase and soon cleared some houses. People were extremely happy to see this. They “begged” the king to buy the wonderful creature for their kingdom. The king readily agreed and bought it for a mule laden with gold. The youngest son happily returned home. Thus, the three sons made the best out of their father’s gifts and lived in wealth and prosperity.

 

Q. 6 Which of the following can be an appropriate title for the story?

A. Bring Out the Devil in You

B. The Rooster and its Crows

C. The Mantra to be Successful – Own a Cat or a Rooster

D. Make the Best of What You Have

E. The Scheming Father

 

Q. 7 Which of the following statements is true in the context of the story?

A. The king was reluctant to purchase the cat from the youngest son

B. All three brothers married three king’s daughters in the end

C. The ailing father had secretly informed the eldest son about making money from his rooster

D. The youngest son was too lazy to work; as a result he went last on his quest for wealth

E. None of the given statements are true

 

Q. 8 Choose the following which is just nearly the opposite in meaning as the word “Right” as used in the story?

A. Dishonest

B. Bad luck

C. Unfitted

D. True

E. Left

 

Q. 9 Which of the following correctly explains the meaning of the phrase “chanced upon” as used in the story?

A. Divided

B. Went with the flow

C. Waited for something

D. Failed to find

E. Came across

 

Q. 10 Which of the following is most nearly the same in meaning as the word “begged” as used in the story?

A. Pleaded with

B. Sued

C. Donated

D. Granted

E. Ordered for

 

Q. 11 As mentioned in the story, people of the island bought the rooster because

A) They genuinely wished to help the eldest son in becoming a merchant.

B) It would help them manage their time more effectively.

C) They are amused by its feature and wanted it as a pet.

A. Only B)

B. Both A) and B)

C. Only A)

D. Only C)

E. Both A) and C)

 

Q. 12 Which of the following is most nearly the opposite in meaning as the word “Certainly” as used in the story?

A. Positively

B. Cautiously

C. Probably

D. Blindly

E. Eccentrically

 

Q. 13 Which of the following characteristics of the father comes across distinctly through the story?

A. He was aggressive

B. He was lonely

C. He rarely became angry

D. He was wise

E. He was an animal lover

 

Q. 14 Which of the following is most nearly the same in meaning as the word “Worthless” as used in the story?

A. Barren

B. Significant

C. Useless

D. Abolished

E. Adopted

 

Q. 15 As mentioned in the story, the father gave gifts to his sons because

A. He didn’t want them to fight in his absence

B. They had demanded those gifts from him

C. Those were the only gifts he could afford at that time

D. He was trying to take revenge on them by giving gifts which were of no use

E. Other than those given as options

 

Questions: 16 – 20

Directions (16-20) :

Rearrange the following five sentences (A), (B), (C), (D) and (E) in proper sequence

to form a meaningful paragraph, then answer the given questions.

(A) Also, crouched on either side of the throne are two enormous beasts known as the Hungry Tiger and the Cowardly Lion.

(B) In the splendid palace of the Emerald City, which is in the centre of the Land of Oz, is a great throne room, where Princess Ozma sits on a throne of glistering emeralds and listen to all the troubles of her people.

(C) So Ozma’s big guards are more ornamental than useful and no one realises that better than the bests themselves.

(D) Around Ozma’s throne, on such occasions, are grouped all the important dignitaries of Oz.

(E) These two beasts are Ozma’s chief guardians, but as everyone loves the beautiful Princess there has never been any disturbances in the great Throne Room.

 

Q. 16 Which of the following should be the FIRST sentence after the rearrangement?

A. A

B. B

C. C

D. D

E. E

 

Q. 17 Which of the following should be the FIFTH (LAST) sentence after the rearrangement?

A. A

B. E

C. D

D. C

E. B

 

Q. 18 Which of the following should be the THIRD sentence after the rearrangement?

A. A

B. B

C. C

D. D

E. E

 

Q. 19 Which of the following should be the FOURTH sentence after the rearrangement?

A. A

B. B

C. C

D. E

E. D

 

Q. 20 Which of the following should be the SECOND sentence after the rearrangement?

A. A

B. B

C. C

D. D

E. E

 

Questions: 21 – 25

Directions (21-25) :

In these questions, sentences with four quoted words are given. One from four words given in quotes may be either wrongly spelt or inappropriate in the context of the sentence. Find out the word which is wrongly spelt or inappropriate. If any, that world will be your answer. If all words given in quotes are correctly spelt and also appropriate in the context of the sentence, then ‘All Correct’ is your answer.

 

Q. 21 The RBI “governor” is the “newly appointed” vice Chairman of the Bank for International Settlements and will “occupy” the post for three years.

A. Governor

B. Newly

C. Appointed

D. Occupy

E. All correct

 

Q. 22 In his new book the author “argues” that firms should “generate value” for “societe” as well as shareholders.

A. Argues

B. Generate

C. Value

D. Societie

E. All correct

 

Q. 23 You do not “need” to be a “customer” of a “specific” bank in order to “purchase” a gift card from that bank.

A. Need

B. Customer

C. Specific

D. Purchase

E. All correct

 

Q. 24 To “deal” with the grievances of taxpayers the department will set up “locale committees” in every “region”.

A. Deal

B. Locale

C. Committees

D. Region

E. All correct

 

Q. 25 The government has “eased foreign” direct investment “norms crossed” 15 sectors.

A. Eased

B. Foreign

C. Norms

D. Crossed

E. All correct

 

Questions: 26 – 30

Directions (26-30) :

In the following passage, there are blanks each of which has been numbered.

These numbers are printed below the passage and against each, five words are suggested, one of which fits the blank appropriately. Find out the appropriate word in each case. The two fish, names Multiwii and Centiwatt lived in a lake with a frog named Uniwit. One evening, as they were sitting by the lake, two fishermen (1) by, planning to come fishing at the lake in the next morning. After (2) the fisherman talk, the frog advised that they should leave the lake (3). Multiwii laughed and said, ‘Don’t be frightened Uniwit. Our many wits say (4) will happen tomorrow.’ Frog, inclined to his instincts and (5) the lake that night. The next morning, fishermen caught Multi Wit and Centiwatt in their nets. And so it is better to pay heed.

 

Q. 26 The word for the blank in (1) is –

A. walking

B. swimming

C. went

D. travel

E. past

 

Q. 27 The word for the blank in (2) is –

A. listening

B. hearing

C. eavesdropping

D. afterwards

E. knowing

 

Q. 28 The word for the blank in (3) is –

A. just

B. closely

C. lately

D. directly

E. immediately

 

Q. 29 The word for the blank in (4) is –

A. nothing

B. this

C. which

D. nobody

E. harm

 

Q. 30 The word for the blank in (5) is –

A. away

B. vanished

C. left

D. hid

E. disappeared

 

Q. 31 Ram invested a certain sum in Scheme A, which offers simple interest at the rate of 12% p.a. for 3 years. He also invested Rs. 6000 in Scheme B, which offers compound interest (compounded annually) at the rate of 10% p.a. for 2 years. If the interest earned from Scheme A is 2/7th of the interest from Scheme B, what is the sum invested in Scheme A?

A. Rs. 10000

B. Rs. 12500

C. Rs. 85000

D. Rs. 13500

E. None of these

 

Q. 32 The sum of two positive numbers is 1840. If 40% of the first number is equal to 75% of the second number, what is the larger number among the two?

A. 1400

B. 1240

C. 1560

D. 1420

E. 1200

 

Q. 33 2/3rd of a number is four more than 1/4th of another number. If the sum of the two numbers is 17, what is their product?

A. 72

B. 64

C. 48

D. 63

E. 92

 

Q. 34 Five years ago, the average of Rimi’s age that time and Ruhi’s age that time was 12 years. Three years from now, if the respective ratio between Rimi’s age and Ruhi’s age that time will be 5:3, what is Rimi’s present age?

A. 17 years

B. 33 years

C. 27 years

D. 19 years

E. 22 years

 

Q. 35 In what respective ratio, wheat of variety A worth Rs. 25 per kg must be mixed with wheat of variety B worth Rs. 18 per kg, so that the new mixture (of both the varieties – A and B) is worth Rs. 20 per kg?

A. 2:5

B. 3:5

C. 3:4

D. 4:3

E. None of these

 

Q. 36 A boat can travel 10.5 km upstream in 42 minutes. If the speed of the water current is 1/6th of the speed of the boat in still water, how much distance (in km( the boat can travel downstream in 48 minutes?

A. 14.8

B. 15.6

C. 16.8

D. 17.4

E. None of these

 

Q. 37 Naveen spends 55% of his monthly salary on rent, EMI and miscellaneous expenses in the respective ratio of 3:4:4. If he spends a total of Rs. 20055 on the rent and EMI together, how much is his monthly salary?

A. Rs. 50300

B. Rs. 49600

C. Rs. 46750

D. Rs. 57300

E. Rs. 56300

 

Q. 38 There are nine positive observations. Average of the first six observations is 10 and of the last four observations is 17. If the average of these nine observations is 13, what is the sixth observation?

A. 17

B. 13

C. 15

D. 19

E. 11

 

Questions: 39 – 43

Directions (39-43) :

What will come in place of question mark (?) in the following number series?

 

Q. 39  16 ? 8 12 24 60

A. 4

B. 10

C. 8

D. 2

E. 12

 

Q. 40  2 3 7 22 89 ?

A. 215

B. 446

C. 268

D. 312

E. 584

 

Q. 41  200 208 192 224 ? 288

A. 160

B. 196

C. 145

D. 154

E. 184

 

Q. 42  188 92 44 20 8 ?

A. 0.2

B. 0.5

C. 2

D. 4

E. 0.4

 

Q. 43  19 36 70 121 ? 274

A. 202

B. 169

C. 189

D. 194

E. 171

 

Q. 44 The selling price of 16 chairs is equal to the selling price of 6 tables. if the total selling price of 5 chairs and 3 tables together is Rs. 780, what is the total selling price of 2 chairs and 1 table together?

A. Rs. 920

B. Rs. 960

C. Rs. 980

D. Rs. 1060

E. Rs. 288

 

Q. 45 Two trains started running from the same point at the same time in opposite directions (one towards North and other towards South). The speed of the two trains is 16 m/s and 14 m/s respectively. How much time will they take to be 378 km apart? (in hours)

A. 2 hours

B. 3 hours

C. 4 hours

D. 3.5 hours

E. 5 hours

 

Questions: 46 – 50

Directions (46-50) : Study the table and answer the given questions.

 

Q. 46 What is the respective ratio between total number of members in health club C in 2009 and 2011 together and that in health club E in the same years together?

A. 13:16

B. 11:16

C. 9:16

D. 13:18

E. 11:18

 

Q. 47 If 62% of the total number of members in health clubs C, D and E together in 2010 are males, what is the total number of male members in health clubs C, D and E together in 2010?

A. 283

B. 280

C. 287

D. 271

E. 279

 

Q. 48 What is the difference between total number of members in health clubs A and B together in 2009 and that in health clubs D and E together in 2013?

A. 83

B. 87

C. 97

D. 93

E. 91

 

Q. 49 What is the average number of members in health clubs A, C and E in 2012?

A. 167

B. 169

C. 177

D. 171

E. 173

 

Q. 50 Number of members in health club B increased by what percent from 2010 to 2013?

A. 48

B. 52

C. 44

D. 54

E. 58

 

Q. 51 The LCM of two numbers is 48. The respective ratio of numbers is 2:3. What is the sum of numbers?

A. 28

B. 32

C. 40

D. 64

E. None of these

 

Q. 52 The perimeter of an equilateral triangle is 21 m more than perimeter of a square. If the respective ratio between the side of the triangle and the side of the square is 9:5, what is the area of the square? (in m²)

A. 25

B. 225

C. 625

D. 144

E. 81

 

Q. 53 A and B can independently finish a piece of work in 25 and 20 days. They started working together and after few days A left. After that B could finish the remaining work in 11 days. After how many days A left the work?

A. 12

B. 13

C. 15

D. 5

E. 10

 

Q. 54 A and B started a business with an investment which is in the ratio 5:2 respectively. After 6 months from the start of the business, C joined with an investment which was twice A’s investment and after 10 months from the start of the business B withdrew all the amount that he invested. If the total annual profit was Rs. 6510, what was the difference between A’s share in the annual profit and B’s share in annual profit?

A. Rs. 1860

B. Rs. 1680

C. Rs. 1580

D. Rs. 860

E. Rs. 1866

 

Questions: 55 – 64

Directions (55-64) :

What will come in place of the question mark (?) in the following questions?

 

Q. 55  50 ÷ 5.5 x 44 + ? = 620

A. 180

B. 175

C. 215

D. 220

E. 225

 

Q. 56   (3+3/5 x 8+1/3) + ? = 4³

A. 26

B. 24

C. 34

D. 36

E. 16

 

Q. 57  (5/9 + 8/5 + 2/5) of ? = 621

A. 261

B. 162

C. 279

D. 216

E. 243

 

Q. 58  40% of 325 + 86 = ?% of 270

A. 75

B. 20

C. 25

D. 70

E. 80

 

Q. 59   (0.15 x 320) ÷ 4 = 3 x 2^?

A. 2

B. 3

C. 1

D. 4

E. 0

 

Q. 60  40% of 620 + 65% of ? = 482

A. 220

B. 260

C. 360

D. 280

E. 240

 

Q. 61  4² x 9² ÷ √324 – 40 = 2^?

A. 4

B. 3

C. 7

D. 5

E. 2

 

Q. 62 Refer image

A. 10

B. 8

C. 12

D. 6

E. 4

 

Q. 63  (453.25 + 157.5 + ?) x 3/4 = 600

A. 161.25

B. 179.25

C. 189.25

D. 182.25

E. 172.25

 

Q. 64 √2916 + √729 = 3^18-?

A. 10

B. 12

C. 14

D. 9

E. 16

 

Q. 65 A circular copper wire of radius 14 cm is bent to form a rectangle. If the breadth and the length of the rectangle are in the ratio 3:8 respectively, what is the length of the rectangle? (in cm)

A. 32

B. 16

C. 24

D. 12

E. None of these

 

Q. 66 In a straight line of twelve persons (facing north), J sits fourth from the right end of the line. Only two persons between J and S. M sits second to the left of S. O sits to the immediate left of M. How many persons sit between O and J?

A. Three

B. Five

C. Cannot be determined

D. None

E. Four

 

Q. 67 In a certain code language READ is written as ‘#5%6’ and PAID is written as ‘$%46’. How is RIPE written in that code language?

A. #4$5

B. #6$5

C. $4#5

D. $4#6

E. Other than those given as options

 

Q. 68 In a certain code language, ‘their new pencils’ is coded as ‘731’. Similarly, ‘box of pencils’ is coded as ‘352’ and ‘their wooden box’ is coded as ‘915’. What will be the code for ‘wooden’ in the given code language?

A. ‘5’

B. ‘9’

C. Either ‘5’ or ‘9’

D. Either ‘1’ or ‘9’

E. ‘1’

 

Q. 69 Pointing to a photograph of a boy Suresh said, “He is the son of the only son of my

mother.” How is Suresh related to that boy?

A. Brother

B. Uncle

C. Cousin

D. Father

 

Q. 70 If A + B means A is the mother of B; A – B means A is the brother B; A % B means A is the father of B and A x B means A is the sister of B, which of the following shows that P is the maternal uncle of Q?

A. Q – N + M x P

B. P + S x N – Q

C. P – M + N x Q

D. Q – S % P

 

Questions: 71 – 75

Directions (71-75) :

Study the following information carefully and answer the questions given below :

Eight persons – A, B, C, D, E, F, G and H – are sitting around a circular table facing the centre but not necessarily in the same order.

• A sits second to the right of H. C sits third to the right of F.

• F is not an immediate neighbour of A.

• Only one person sits between F and G.

• B is an immediate neighbour of G.

• D sits second to the left of B. D is not an immediate neighbour of G.

 

Q. 71 Four of the following five are alike in a certain way based on their positions in the given arrangement and so form a group. Which is the one that does not belong to the group?

A. DCF

B. GAB

C. CDH

D. BEG

E. HFE

 

Q. 72 Which amongst the following statements is true regarding E, as per the given Arrangement?

A. None of the given statements is true

B. E is one of the immediate neighbours of B

C. D sits second to the right of E

D. C sits to the immediate left of E

E. Only three persons sit between E and G

 

Q. 73 Who among the following represent the immediate neighbours of C?

A. H, A

B. A, D

C. B, D

D. D, G

E. H, G

 

Q. 74 Who among the following sits second to the right of B?

A. C

B. E

C. A

D. H

E. F

 

Q. 75 How many persons are seated between D and H, when counted from the left of D?

A. Three

B. One

C. Four

D. Two

E. None

 

Questions: 76 – 80

Directions (76 -80) :

In each of the following questions, a group of numbers/symbols followed by five combinations of letter codes is given. You have to find out which of the combinations correctly represents the group of numbers/symbols based on the given coding system and the conditions and mark that combination as your answer.

Conditions :

(i) If the first and the last elements are odd numbers then their codes are to be interchanged

(ii) If an even number is immediately followed as well as immediately preceded by a symbol then that even number is to be coded as ‘O’

(iii) If the third element is symbol then the first element is to be coded as the code of that symbol

(iv) If the second last element is an odd number then the code of that odd number is to be interchanged with the code of last element.

(Please note : All the elements have to be counted from left to right to fulfil the conditions)

 

Q. 76 The combination is 97@★+4 –

A. WZGXVU

B. UZGXDW

C. UZGXVW

D. GZGXVW

E. GZGVXW

 

Q. 77 The combination is 7#68&3 –

A. YADCKZ

B. YAJCKZ

C. ZAJCKY

D. KAJCKY

E. ZAJCYK

 

Q. 78 The combination is 4#5%92 –

A. PAWHUB

B. WAOHBU

C. BAPHUW

D. BAPWUH

E. WAPHBU

 

Q. 79 The combination is 754%2$ –

A. ZPWHBE

B. HPWHBE

C. ZPWHOE

D. HPWBOE

E. EPWHBZ

 

Q. 80 The combination is $24@8★ –

A. EGBWGCX

B. XBOGCE

C. EBWGOX

D. WBEGCX

E. XBWGCE

 

Questions: 81 – 85

Directions (81-85) :

Study the following information carefully and answer the questions given below :

Seven persons – E, F, G, H, M, N and O – are seated in a straight line facing north, with equal distances between each other but not necessarily in the same order.

• He sits third to the right of O. Neither H nor O sits at any of the extreme ends of the line.

• G sits second to the left of N. N is not an immediate neighbour oh H.

• Only three persons sit between N and E.

• F is not an immediate neighbour of E.

 

Q. 81 How many persons are seated between O and F?

A. Four

B. Two

C. Three

D. One

E. None

 

Q. 82 Who amongst the following sits exactly in the middle of the line?

A. E

B. F

C. G

D. O

E. H

 

Q. 83 Which of the following represents persons seated at the two extreme ends of the line?

A. F, G

B. E, F

C. E, M

D. F, N

E. E, G

 

Q. 84 What is the position of F with respect to E?

A. Third to the left

B. Third to the right

C. Second to the right

D. Fourth to the left

E. Fifth to the right

 

Q. 85 Who amongst the following sits immediate right of M?

A. O

B. G

C. E

D. H

E. F

 

Questions: 86 – 90

Directions (86-90) :

In each of the following questions, relationship between different elements is shown in the statements. The statements are followed by two conclusions numbered I and II. Study the conclusions based on the given statement(s) and mark the appropriate answer.

 

Q. 86 Statement – L≤V≤O = D

Conclusions –

I. D=L

II. L

A. Both the conclusion I and II are true

B. Either conclusion I or conclusion II is true

C. Neither conclusion I nor conclusion II is true

D. Only conclusion I is true

E. Only conclusion II is true

 

Q. 87 Statement – J=C > Y=K Conclusions –

I. X>C

II. K

A. Both the conclusion I and II are true

B. Either conclusion I or conclusion II is true

C. Neither conclusion I nor conclusion II is true

D. Only conclusion I is true

E. Only conclusion II is true

 

Q. 88 Statement – R>E≥T = I≥M

Conclusions –

I. R>I

II. E≥M

A. Both the conclusion I and II are true

B. Either conclusion I or conclusion II is true

C. Neither conclusion I nor conclusion II is true

D. Only conclusion I is true

E. Only conclusion II is true

 

Q. 89 Statement – OConclusions –

I. OII. P>N

A. Both the conclusion I and II are true

B. Either conclusion I or conclusion II is true

C. Neither conclusion I nor conclusion II is true

D. Only conclusion I is true

E. Only conclusion II is true

 

Q. 90 Statement – H≤AW;S = B

Co nc lus io ns –

I. S >H

II. W

A. Both the conclusion I and II are true

B. Either conclusion I or conclusion II is true

C. Neither conclusion I nor conclusion II is true

D. Only c o nc lus io n I is true

E. Only conclusion II is true

 

Q. 91 How many such pairs of letters are there in the word CHUNKS each of which has as many letters between them in the word (in both forward and backward directions) as they have between them in the English alphabetical series?

A. Three

B. One

C. Two

D. No ne

E. More than three

 

Q. 92 In a certain code language FLUTE is coded as HJWRG and GIANT is coded as IGCLV. In the same code language, how will PLOTS be coded as?

A. RJMV U

B. NUMRQ

C. QMP U T

D. NNMV Q

E. RJQRU

 

Q. 93 If all the numbers in 67452893 are arranged in ascending order from left to right, the position (s) of how many number (s) will remain unchanged?

A. No ne

B. One

C. Two

D. Thr ee

E. Mo r e t han t hr ee

 

Directions (94-95) :

Study the following information carefully and answer the questions given below :

Five persons – L, M, N, O and P – have different typing speeds, M types faster than L, but slower than P. O types slower than only one person. L does not type at the slowest speed. The one who types the third fastest types 73 words per minute. The one who type the slowest types 29 words per minute.

 

Q. 94 Who among the following types at the second slowest speed?

A. L

B. P

C. O

D. M

E. N

 

Q. 95 If P types 12 more words per minute than M, then how many words does P type per minute?

A. 65

B. 85

C. 61

D. 41

E. 45

 

Questions : 96 – 100

Directions (96-100) :

The following questions are based on five three digit numbers given below :

395 432 823 657 278

 

Q. 96 The positions of the first and the second digits of each of the numbers are interchanged. What will be the resultant if first digit of highest number thus formed is divided by the third digit of the lowest number thus formed?

A. 1.5

B. 1

C. 4

D. 3

E. 2.5

 

Q. 97 If ‘2’ is subtracted from the third digit of every odd number and ‘1’ is added to the second digit of every even number, in how many numbers will a digit appear twice?

A. Two

B. Three

C. Four

D. One

E. None

 

Q. 98 What will be the resultant if third digit of the lowest number and second digit of the highest number are multiplied?

A. 21

B. 4

C. 54

D. 12

E. 16

 

Q. 99 If all the numbers are arranged in ascending order from left to  right, which of the following will be sum of all three digits of the number which is fourth from  the left?

A. 133

B. 18

C. 9

D. 17

E. 15

 

Q. 100 If in each number all the digits are arranged in ascending order from left to right within the number, how many numbers thus formed will be even numbers?

A. Three

B. No ne

C. Four

D. One

E. Two

 

 

Answer Sheet
Question 1 2 3 4 5 6 7 8 9 10
Answer E B C C E D E C E A
Question 11 12 13 14 15 16 17 18 19 20
Answer A C D C E B D A D D
Question 21 22 23 24 25 26 27 28 29 30
Answer D D E B D C C E A C
Question 31 32 33 34 35 36 37 38 39 40
Answer A E A E A C D E C B
Question 41 42 43 44 45 46 47 48 49 50
Answer A C C E D A E C E B
Question 51 52 53 54 55 56 57 58 59 60
Answer C B D A D C E E A C
Question 61 62 63 64 65 66 67 68 69 70
Answer D D C C A B A B D C
Question 71 72 73 74 75 76 77 78 79 80
Answer B C E A D D B E A C
Question 81 82 83 84 85 86 87 88 89 90
Answer D B E A C B E A C D
Question 91 92 93 94 95 96 97 98 99 100
Answer B E C A E D E E B A
×

Hello!

Click one of our representatives below to chat on WhatsApp or send us an email to info@vidhyarthidarpan.com

×